Calculus
About this book : http://matrix.skku.ac.kr/Cal-Book/
http://matrix.skku.ac.kr/Cal-Book1/
http://matrix.skku.ac.kr/Cal-Book1/Ch1/
http://matrix.skku.ac.kr/Cal-Book1/Ch2/
http://matrix.skku.ac.kr/Cal-Book1/Ch3/
http://matrix.skku.ac.kr/Cal-Book1/Ch4/
http://matrix.skku.ac.kr/Cal-Book1/Ch5/
http://matrix.skku.ac.kr/Cal-Book1/Ch6/
http://matrix.skku.ac.kr/Cal-Book1/Ch7/
http://matrix.skku.ac.kr/Cal-Book1/Ch8/
...
http://matrix.skku.ac.kr/Cal-Book1/Ch13/
http://matrix.skku.ac.kr/Cal-Book1/Ch14/
http://matrix.skku.ac.kr/Cal-Book1/Ch15/
Chapter 13. Partial Derivatives
13.1 Multivariate Functions
문제풀이 by 구본우 http://youtu.be/As_0AYApHlM
13.2 Limits and Continuity of Multivariate Functions
13.3 Partial Derivatives http://youtu.be/LR89Ct3cEDY
문제풀이 by 김동윤 http://youtu.be/rSYLp1mSMXY
13.4 Differentiability and Total Differentials
문제풀이 by 김범윤 http://youtu.be/qDmCWBiXbIA
13.5 The Chain Rule http://youtu.be/r3dGYL1vkEU
문제풀이 by 김유경 http://youtu.be/vzN5By6qzvM
13.6 Directional Derivatives and Gradient http://youtu.be/o8L_ShRANjo
문제풀이 by 김태현 http://youtu.be/2_7TOUuzJoE
13.7 Tangent Plane and Differentiability http://youtu.be/uOf-5YHKGI4
문제풀이 by 서용태 http://youtu.be/GDkE8OqUvsk
13.8 Extrema of Multivariate Functions http://youtu.be/oDZUkOEszOQ
문제풀이 by 오교혁 http://youtu.be/FWmk_MasIjE
13.9 Lagrange Multiplier
문제풀이 by 이원준 http://youtu.be/YMGdQWBzyrI
13.1 Multivariate Functions
Multivariate Functions
So far we have discussed calculus of functions of one variable. In this section we deal with functions of two or more variables. However in real life a function depends upon many dependent variables. Such functions, in general are called multi-variable functions or functions of several variables. In this section, we primarily deal with functions of two variables. All the concepts can be analogously extended to functions of two or more variables. Many functions in real life depend upon more than one variables. In chapters 3 and 5, we discussed how to obtain the derivatives and integrals of functions with one independent variable. In this section, we will consider functions with two or more independent variables.
Let us look at some examples: (i) The area of a rectangle depends upon the length
and width
of the rectangle. Therefore, we can think of area as function of two variables,
. (ii) The temperature
of particular place on earth depends on the longitude
and the latitude
of that point. So we can think of temperature as a function of two variables, say
. (iii) The cost of a mobile hand-set depends upon many factors.
In this section we deal with real valued functions of two real variables. The domain of such functions is a set of points in . We will assume that
is endowed with Euclidean distance, that is distance between any two points,
and
is given by
.
A point in can also be referred to as a vector, which is the position vector of that point.
A real valued function of two real variables in general can be defined as follows:
DEFINITION1
Let . A function
of two variables is a rule that assigns to each point
a unique real number denoted by
. The set
is called the domain of
and its range is the set of values that
takes.
Generally, we write to make explicit the value taken on by
at the point
. Here
and
are called independent variables and
is called the dependent variable.
Most often the domain of the function is not mentioned explicitly, however it can be obtained from the function’s expression by simply looking at set of points on which the function is well defined. Let us look at some examples.
EXAMPLE1
Find the domain of the following function:
(a)
(b)
(c)
(d)
(e) .
Solution. (a) Note that the right handside of the expression works for all values except when the denominator is zero. That is, the domain of this function is
.
(b) The domain of this function is set of points such that
. That is,
. Thus the domain
of
is
.
(c)The domain of is a set of points
such that
. See Figure 1. That is
.
Figure 1
(d)
We know that is defined only for positive reals. That is
. This happens when
(i),
or (ii)
,
. For (i),
.
For (ii), .
Then the domain of
is
. Thus
. See Figure 2.
Figure 2
(e)The domain of this function is when right handside make sense. That when and
. The domain is plotted in Figure 3.
Figure 3
■
Graph of a Multivariate Function
Let . Then the graph of
is a set of points
graph .
Usually, the graph of a function is called a surface in
. (See Figure 4.) In Figure 5 the surface is the graph of the polynomial function
is plotted.
Figure 4 Graph of a function of and
is a surface Figure 5 Graph of a polynomial function
Unlike one variable function , plotting the graph of a function
in
is not an easy task. One way to plot is to look at set of points
for various values of
, which is the intersection of the surface and the plane
. The intersection is called the level curve at the level
. If we know these level curves for all values of
, then we can visualize the surface
.
Note that level curve is the set of all points in the domain of
at which
takes value
. Level curves are also called contour lines. Thus to plot the graph of a surface
all we need is to plot level curves (contour lines) for various values of
and then put these level curves at the corresponding level
planes. (See Figure 6(a).)
A contour map of a function is a 2-dimensional graph showing several level curves corresponding to several values of
. A contour map of
lies in the
-plane. (See Figure 6(b).)
Figure 6 Surface in (a) and level curves in (b)
EXAMPLE2
Plot the surface and level curves of .
■
Figure 7 Surface and level curves in Example 2
In most instances the task of graphing level curves of a function of two variables is formidable. A CAS was used to generate the surfaces and corresponding level curves in Figure 8.
(a) (b)
Figure 8 Graph of in (a); level curves in (b)
The level curves of a function are also called contour lines. On a practical level, contour maps are often used to display curves of equal elevation. In Figure 9, we see that a contour map illustrates the various segments of a hill that have a given altitude. This shows heights of the mountain in in Figure 10, which show the volcano Mt. Halla, in the state of Jeju, Korea.
Figure 9 Contour map of Mt. Halla Figure 10 Map of Mt. Halla in the state of Jeju, South Korea
CAS. Graph the level curves of the following functions along with the surface:
(1)
(2)
(3)
(4)
(1) http://matrix.skku.ac.kr/cal-lab/m-Sec13-1-Exm-3.html
[CAS] http://sage.skku.edu/ 또는 https://sagecell.sagemath.org/
var('x, y')
f(x, y)=x^2+y^2
contour_plot(f(x, y), (x, -2, 2), (y, -2, 2), fill=False, cmap='hsv', axes=True, labels=True)
Figure 11
(2)
var('x, y')
f(x, y)=x^2-y^2
contour_plot(f(x, y), (x, -2, 2), (y, -2, 2), fill=False, cmap='hsv', axes=True, labels=True)
Figure 12
(3) [CAS] http://sage.skku.edu/ 또는 https://sagecell.sagemath.org/
var('x, y')
f(x, y)=sin(x)*cos(y)
contour_plot(f(x, y), (x, -2, 2), (y, -2, 2), fill=False, cmap='hsv', axes=True, labels=True)
Figure 13
(4) Level curves along with the surface:
var('x, y')
f(x, y)=4*x*y*exp(-x^2-y^2)
contour_plot(f(x, y), (x, -2, 2), (y, -2, 2), fill=False, cmap='hsv', axes=True, labels=True)
Figure 14
■
EXAMPLE 4
Graph and plot the level curves
,
and
in the domain of
in the plane.
Solution. The domain of is the entire
-plane, and the range of
is the set of real numbers less than or equal to
. The graph is the paraboloid
, a portion of which is shown in Figure 15. In Figure 16, The level curve
is the set of points in the
plane such that
.
This is the circle of radius centered at the origin. Similarly, the level curves
and
are the circles
.
The level curve consists of the origin alone. ■
Figure 15 Figure 16
Functions of Three Variables
The definition of functions of three or more variables are simply generalizations of the definition of one variable. For example, a function of three variables is a rule of correspondence that assigns to each ordered triple of real number in a subset of
, one and only one number
in the set
of real numbers. A function of three variables is usually denoted by
or
.
For example, the volume and surface area S of a rectangular box are polynomial functions of three variables:
and
.
The set of points in space where a function of three independent variables has a value
is called a level surface of
.
EXAMPLE 5
Describe the level surfaces of the function .
Solution. The value of is the distance from the origin to the point
. Each level surface
is a half-sphere of radius
centered at the origin. The level surface
consists of the origin alone. ■
We are not graphing the function in this example. We are looking at level surfaces in the domain of the function. The level surfaces show how the function values change as we move through its domain . If we remain on a sphere of radius
centered at the origin, the function maintains a constant value, namely
. If we move from a point on one sphere to a point on another, the function value increases if we move away from the origin and decreases if we move toward the origin. The manner in which the values change depend on the direction chosen; the relationship between the function values and the direction travelled is important.
EXAMPLE6
(a) The level surfaces of the polynomial are a family of parallel planes defined by
. (See Figure 18.)
Figure 18 Level surfaces in (a) of Example 6
(b) The level surfaces of the polynomial are a family of concentric spheres defined by
,
. See Figure 17.
Figure 17 Level surfaces in (b) of Example 6
(c) The level surfaces of the rational function are given by
or
. A few members of this family of paraboloids are
given in Figure 19.
Figure 19 Level surfaces in (c) of Example 6
(a) http://matrix.skku.ac.kr/cal-lab/m-Sec13-1-Exm-6.html
[CAS] http://sage.skku.edu/ 또는 https://sagecell.sagemath.org/
var('x,y,z')
f(x,y,z)=x-y+2*z
p=Graphics()
for k, col in [(1, 'red'),(2, 'orange'),(3, 'yellow'),(4, 'green')]:
p+=implicit_plot3d(f(x,y,z)==k,(x,-2,2),(y,-2,2),(z,-1,3),opacity=0.7, color=col)
p
(b)
var('x,y,z')
f(x,y,z)=x^2+y^2+z^2
p=Graphics()
for k, col in [(1, 'red'),(2, 'orange'),(3, 'yellow'),(4, 'green')]:
p+=implicit_plot3d(f(x,y,z)==k,(x,-2,2),(y,-2,2),(z,-3,1),opacity=0.7, color=col)
p # Figure 17
(c)
var('x,y,z')
f(x,y,z)=(x^2+z^2)/y
p=Graphics()
for k, col in [(1, 'red'),(2, 'orange'),(3, 'yellow'),(4, 'green')]:
p+=implicit_plot3d(f(x,y,z)==k,(x,-2,2),(y,-2,2),(z,-3,3),opacity=0.7, color=col)
p
■
13.1 EXERCISES (Multivariate Functions)
http://matrix.skku.ac.kr/Cal-Book/part2/CS-Sec-13-1-Sol.html http://youtu.be/As_0AYApHlM
1. If , find (a)
, (b)
, (c)
.
(a) .
(b) .
(c)
.
2. , find
(a) , (b)
.
(a) .
(b) .
3-4. Find the domain of the given function.
3.
4.
5-6. Find the range of the given function.
5.
[CAS] http://sage.skku.edu/ 또는 https://sagecell.sagemath.org/
var('x,y,z')
implicit_plot3d(1+exp(x*y)==z,(x,-2.5,2.5),(y,-2.5,2.5),(z,0,3),opacity=0.3)
Then the range is .
6.
7-8. Sketch typical level surfaces of the function.
7.
[CAS] http://sage.skku.edu/ 또는 https://sagecell.sagemath.org/
var('x,y,z')
f(x,y,z)=y+z
p=Graphics()
for k, col in [(1,'red'), (2,'orange'), (3, 'yellow'), (4,'green')]:
p+=implicit_plot3d(f(x,y,z)==k,(x,-2,2), (y,-2,2),(z,-1,3),opacity=0.7,color=col)
p
8.
9.
10. Given that
find the ,
and
intercepts of the level surface that passes through
.
13.2 Limits and Continuity of Multivariate Functions
Limits and Continuity
Before discussing limits and continuity, we need to introduce some terminology about sets that will be useful in the sections and chapters that follow. The set
is called an open disk centered at with radius
. On the other hand, the set
is a closed disk. A closed disk contains all points interior to and on a circle. See Figure 1 (a). Let be some region of the
-plane. Then a point
is said to be an interior point of
, if there is at least one open disk centered at
that contains only points of
. We say that
is a boundary point of
if the interior of each open disk centered at
contains points from both
and
. The region
is said to be open if it contains no boundary points and closed if it contains all its boundary points. See Figure 1. A region
is said to be bounded if it can be contained in a sufficiently large rectangle or disk in the plane. These concepts can be carry over naturally to
. For example, an open ball consists of all points interior to, but not on, a sphere with center
and radius
;
.
A region in is bounded if it can be contained in a sufficiently large disk.
Figure 1 Various regions in
Let be a function defined on a region
.
which means is getting close to the number
as
close to
. A precise definition can be given as follows;
DEFINITION1
Let and
(The point
need not necessarily be in
). Then we say that the limit of
as
approaches to
is
, if for every
, there exists
such that such that
when
for
.
This can be also expressed as
and
as
.
EXAMPLE1
Let . Let us check if
exists.
Solution. It is straightforward to guess that the limit exists and it is . Let
be given. To prove that
, we need to find
such that when
, we have
. Note that
for all
. Hence any choice of
will work. Thus
. ■
EXAMPLE2
Let . Let us check if
exists.
Solution. It is not hard to guess that when ,
. Therefore, the limit must be
. Let us prove this. Let
. We need to find
such that when
,
we have .
For any , it is easy to that (See Figure 2.)
Figure 2
and
. Thus we have
(Triangle Law of Modulus)
.
Therefore,
.
This suggests that we may choose . Now if
, from
we have
. Hence,
exists and it is
. ■
EXAMPLE3
(1) Let . Then show that
.
(2) Let . Then show that
.
Solution. (1) Let . We need to find
such that when
,
we have .
For any , it is easy to that
.
This suggests that we may choose . Now if
, from
we have
. Hence,
.
(2) Similar to (1), we can prove . ■
Limit Along a Path
The notion of “approaching” a point
is not as simple as it is for functions of one variable. In the
-plane, there are an infinite number of ways of approaching a point
, as shown in Figure 3. In order that
exist, we require that
approach to
along every possible curve or path through
. In particular,
(ⅰ) If does not approach the same number
for two different paths to
, then
does not exist.
(ⅱ) If the limit of does not exists along some path through
, then limit
at
does not exist.
Figure 3
In the discussion of that follows we shall assume that the function
is defined at every point
in an open disk centered at
but not necessarily at
itself.
EXAMPLE4
Investigate the limit of
at
.
Solution. Let and
in such a way that
. Then along this line
.
Since the limit of the function depends on the manner in which approach to
, the function
does not have a limit at
. For example, for
,
and for
,
. ■
EXAMPLE5
Find the limit, if it exists, of the following function as .
(a)
(b)
Solution. (a) Since and
,
we have .
Note that iff
and
.
Thus, as
, and so
.
That is, .
(b) Let and
in such a way that
. Then along this line
Since the limit of the function depends on the manner in which approach to
, the function
does not have limit at
. For example, for
,
and for
,
. ■
Finding the limit of a function directly from the definition is not an easy task. The following are properties of limit of two variable functions which are similar to properties of one variable functions. These properties give a tool to compute limit of various functions.
THEOREM 2 The Power Rule
Let and
be real numbers and suppose that
,
. Then
1. ,
2.
3.
4.
5. , if
6.
7. If and
are integers,
and
have no common factors and
, then
,
where we have if
is even and
and
are integers.
EXAMPLE6
Using the laws of limit in Theorem 2, find the limit of
(1) at
.
(2) .
Solution. (1) From Theorem 2, we have we have
.
(2) From Theorem 2, we have
.
Therefore, . ■
EXAMPLE7
Find the limit of the following function as .
(a) (b)
Solution. (a) Let . Then one can show that
if and only if
. We have
.
Therefore,.
(b)When the point is on the straight line
,
. Therefore, the limit of
is
as
approaches
along line
. This limit changes as
changes. Hence,
does not have a limit as
. ■
EXAMPLE8
Find the limit of the following functions as .
(a) (b)
Solution. (a) Since, , we have
Note that iff
and
Thus, as
, and so
.
That is, .
(b) Letting along any non-vertical line through the origin. Let
along the
, we have
.
Therefore, depending on the value of , different paths will lead to different values of the limit as
. Hence the limit does not exist as
. ■
Figure 4 Figure 5
EXAMPLE9
Find .
http://matrix.skku.ac.kr/cal-lab/m-Sec13-2-Exm-9.html
[CAS] http://sage.skku.edu/ 또는 https://sagecell.sagemath.org/
Solution.
var('x,y')
f(x,y)=(x-2*y)/(x+3*y)
limit(limit(f,x=1),y=2)
Answer : (x, y) |--> -3/7
var('x,y')
f(x,y)=(x-2*y)/(x+3*y)
limit(limit(f,y=2),x=1)
Answer : -3/7 ■
THEOREM 3
For a function in neighborhood of point
, suppose that
exist. Let
.
Then and
converges to
. Furthermore,
and
can be distinguished by denoting the former as the double limit while the latter as the successive limit. Since
is practically a limit of single variable function, we just repeats the limit process twice.
Proof. Since , for any given
, there exists a number
such that if
then
.
With the same argument, since , for any given
, there exists a number
such that
if then
.
Since , for any given
, there exists a number
and
such that
if then
.
Thus, if , then
and simultaneously if
, then
.
This implies
.
Likewise, this may be proven in a similar manner for limits taken in varying orders, that is, first , then
and first
, then
. ■
In Example 7 (b) and
. Thus
and
exist but two limits are not equal. Then
does not exists.
The function is one of examples where the converse of Theorem 3 is false. For this function,
if , then
and
if , then
.
Therefore,
.
However, when , respectively
. Thus in any domain near
, no matter how small, there are points that are assigned
and points that are assigned
by the function
. This means that the limit of the function at
does not exists. The
may not exist, even if
is satisfied.
EXAMPLE10
Using Sage, find the limit of
(1) at
.
(2) .
(3) Find the example of .
Solution.
(1) http://matrix.skku.ac.kr/cal-lab/m-Sec13-2-Exm-10.html
[CAS] http://sage.skku.edu/ 또는 https://sagecell.sagemath.org/
var('x, y')
f(x, y)=(3*x+y^2+4*x*y)
limit(f(x, y), x=1)
y^2 + 4*y + 3
limit(f(1, y), y=1)
Answer : 8
(2)
var('x, y')
f(x, y)=(x^2-4*y+1)/(x+2*y+3)
limit(f(x, y), x=0)
-(4*y - 1)/(2*y + 3)
limit(f(0, y), y=0)
Answer : 1/3
(3) Let . We have
,
.
Then since , a function
is an example of (3). ■
Continuity of Functions of Two Variables
The following definition of continuity for function of two variables is analogous to the definition of continuity for a one variable function.
DEFINITION4
Let and
. Then
is said to be continuous at
if
. That is,
exists and it is equal to the value of the function. If
is continuous at every point in
, then we say that
is continuous on
.
EXAMPLE11
It is easy to see that functions defined in Examples 1, 2 and 3 are continuous on . The proof is similar to the solutions given in Examples 1, 2 and 3. ■
EXAMPLE12
Determine the points at which the following function is continuous.
http://matrix.skku.ac.kr/cal-lab/m-Sec13-2-Exm-12.html
Solution. The function is a rational function, so it is continuous at all points of its domain, which consists of all points of
except
. In order for
to be continuous at
, we must show that
.
You can verify that as approaches
along paths of the form
, where
is any constant, the function values approach
. Now consider parabolic paths of the form
, where
is a nonzero constant. (See Figure 6.)
Figure 6
For the choice of , notice that if
is replaced by
in
, the result involves the same power of
(in this case,
) in the numerator and denominator were canceled.
This time we substitute and note that
as
;
We see that along parabolic paths, the limit depends on the approach path. For example, with , along the path
, the function values approach
. (See Figure 7.) Because function values approach two different numbers along two different paths, the limit at
does not exist, and
is not continuous at
.■
Figure 7
EXAMPLE13
Investigate the continuity of
at
.
Solution.
,
Hence it is continuous at . ■
Properties of Continuous Functions
Using the properties of limits in Theorem 2, one can show that sums, differences, products and quotients of continuous functions are continuous on their domains under appropriate conditions. For example:
(i) Any polynomial function of two variables and
is continuous on
. Note that a polynomial of
and
is a function which is obtained by adding terms of the form
where
and
are non negative integers.
(ii) A rational function is continuous on
as numerator and denominator are continuous on
and the denominator satisfies
for all
.
THEOREM 5 Continuity of Composite Functions
If is continuous at
and
is continuous at
, then the composite function
is continuous at
.
EXAMPLE14
Investigate the continuity of ,
.
Solution. Since is continuous for all
and
is continuous for all
,
is also continuous for all
by using Theorem 5. Similarly, since
is continuous for all
and
is continuous for all
,
is also continuous for all
by using Theorem 5. ■
The following theorem is an important concept which is used in quite often while dealing with continuous functions. It says that if is a continuous function and has a non zero value at a point
, then it continues to be non zero near
in the domain of
.
THEOREM 6
If is a continuous function at the point
and
, then
has the same sign as
near the point
.
Continuity of Functions of Three Variables
The notions of limit and continuity for functions of three or more variables are natural extensions of those just considered. For example, a function is continuous at
if
.
The polynomial function is continuous throughout
. The rational function
is continuous except at the single point . The rational function
is continuous except at points on the plane
.
13.2 EXERCISES (Limits and Continuity of Multivariate Functions)
http://matrix.skku.ac.kr/Cal-Book/part2/CS-Sec-13-2-Sol.html
1-7. Find the limit, if it exists, or show that the limit does not exist.
1.
Solution. Let and
along the line
.
Then.
For example, for ,
and
for
. Therefore the limit does not exists.
2. .
http://matrix.skku.ac.kr/cal-lab/cal-12-1-1.html
Solution. .
[CAS] http://sage.skku.edu/ 또는 https://sagecell.sagemath.org/
var('x,y')
f(x,y)=x^3/(2*x^2+6*y^4)
limit(limit(f(x,y),x=0),y=0)
0
limit(limit(f(x,y),y=0),x=0)
0
The limit exists and it is .
3.
http://matrix.skku.ac.kr/cal-lab/cal-12-1-Exs-3.html
[CAS] http://sage.skku.edu/ 또는 https://sagecell.sagemath.org/
var('x, y')
f(x, y)=(x^2-y^2)/(x^2+y^2)
limit(f(x, 0), x=0)
1
limit(f(0, y), y=0)
-1
Answer : The limit does not exist.
4. .
http://matrix.skku.ac.kr/cal-lab/cal-12-1-3.html
After plotting the function, we know it converges. So the order in and
does not effect.
[CAS] http://sage.skku.edu/ 또는 https://sagecell.sagemath.org/
var('x, y')
f(x, y)=(sin(x^3 + y^3))/(x^2+y^2)
limit(f(x, y), x=0)
sin(y^3)/y^2
limit(f(0, y), y=0)
Answer : 0
5. .
http://matrix.skku.ac.kr/cal-lab/cal-12-1-4.html
Let and
along the line
.
[CAS] http://sage.skku.edu/ 또는 https://sagecell.sagemath.org/
var('x,y')
f(x, y)=(sin(x^3 + y^3))/(x^2+y^2)
limit(limit(f(x,y),x=0),y=0)
y
limit(limit(f(x,y),x=0),y=0)
0
Answer : The limit exists and it is .
6. .
After plotting the function, we know it converges. Let and
along the line
.
Answer : 0
7. .
http://matrix.skku.ac.kr/cal-lab/cal-12-1-6.html
[CAS] http://sage.skku.edu/ 또는 https://sagecell.sagemath.org/
var('x, y, z')
f(x, y)=x*y*z/(x*y+y*z+z*x)
limit(f(x, y, z), x=0)
0
So
show(limit(f(0, y, z), y=0))
show(limit(f(0, y, z), z=0))
0
0
Answer : The limit exists and it is .
8. What value of for
will
make the function continuous at .
Solution. ⇒
Since , then
by Squeeze Theorem.
Hence, if we define , then
will be continuous at
.
9-10. Let each of the following functions have the value 0 at the origin. Which of them are continuous at the origin? Explain your answer.
9. .
http://matrix.skku.ac.kr/cal-lab/cal-12-1-7.html
[CAS] http://sage.skku.edu/ 또는 https://sagecell.sagemath.org/
var('x, y')
f(x, y)=(x*2*y^2)/(x^2+y^4)
limit(f(x, y), x=0)
0
limit(f(0, y), y=0)
0
The limit is . Hence, if we define
, then
is continuous at the origin.
10.
http://matrix.skku.ac.kr/cal-lab/cal-12-1-9.html
[CAS] http://sage.skku.edu/ 또는 https://sagecell.sagemath.org/
var('x, y, z, w')
f(x, y, z, w)=(x^2+y^4+z^2*w^2)/(x^4+y^2+z^2+w^2)
limit(f(x, 0, 0, 0), x=0)
+Infinity
The limit does not exist.
11-12. Find and the set on which
is continuous.
11. ,
.
Solution. Since is continuous for all
and
is continuous for all
,
is continuous on a set
by using Theorem 5 in Section 13.2.
12. ,
.
Solution. Since is continuous for all
and
is continuous for all
,
is continuous on a set
by using Theorem 5 in Section 13.2.
13.3 Partial Derivatives
In this section, we will deal with partial derivatives, which is an important concept in multivariate calculus. Partial derivatives are (essentially) derivative of one variable functions. Consider defined on a domain,
. Let us fix a point
. Suppose we fix one of the variables, say
, then
becomes a function of one variable
. Let
. If
is differentiable at
, that is,
exists, then we say that has a partial derivative at
with respect to
. Similarly we can define the partial derivative of
with respect to
at
. See Definition 1 for a generalization of this idea.
First-Order Partial Derivatives
DEFINITION1
Let be a function of two variables
and
.
The partial derivative of with respect to
is defined as
, provided this limit exists.
The partial derivative of with respect to
is defined as
, provided this limit exists.
The partial derivative of with respect to
may be denoted by (any of the following)
.
Similarly, the partial derivative of with respect to
may be denoted by
.
Geometric Meaning of the Partial Derivative
Let us consider the function . Let
be a point on the surface where
. We wish to look at the geometric meaning of
and
. To find
, we look at the curve of intersection of the surface
and the plane
, parallel to the
plane. This is the same as saying fix
in
. Then
is the slope of the tangent to the curve of intersection at
as the rate change of
for
at this point. (See Figure 1.)
Figure 1 Intersection of the surface with the plane
Similarly, is the slope of the tangent of the curve at the point
as the rate change of
for
at this point. (See Figure 2.)
Figure 2 Intersection of the surface with the plane
EXAMPLE1
Find partial derivatives with respect to and
for the following functions at the given point.
(a) .
(b) .
(c) .
Solution. (a) .
Thus ,
.
(b) The partial derivatives are
,
.
Then ,
.
(c) Let . Then
Then and
. ■
EXAMPLE2
Show that for
.
Solution. The partial derivatives are
and
.
Therefore, . ■
Functions of Three Variables
Let be a function of three variables
and
. The partial derivatives of
with respect to
, with respect to
, and with respect to
are defined as follows
provided the limits on the right hand side exist.
In general, if is a function of
variables, then the partial derivative of
with respect to the
variable,
, is defined to be
.
To compute , we differentiate with respect to
while holding the remaining
variables fixed.
EXAMPLE3
Find ,
and
if
.
Solution. ,
and
.
var('x,y,z')
f=exp(x*y)*log(z)
fx=diff(f,x);fy=diff(f,y);fz=diff(f,z)
print fx
print fy
print fz
Answer : y*e^(x*y)*log(z), x*e^(x*y)*log(z), e^(x*y)/z. ■
Second and Higher Order Partial Derivatives
Suppose that has first order partial derivatives
and
. Note that
and
are also function of
and
. Therefore, we can talk about their partial derivatives if they exist. If
and
have partial derivatives with respect to
and
, then the following four possibilities exist:
,
,
,
.
These are called second order partial derivatives of .
and
are called mixed second order partial derivatives. In general, the second order mixed partial derivatives need not be equal.
Third order partial derivatives may also be defined, for instance
.
.
.
The higher partial derivative may be defined similarly if they exist.
EXAMPLE4
Let
be a function of two variables .
Verify that and
are not define at
.
Solution.
[CAS] http://sage.skku.edu/ 또는 https://sagecell.sagemath.org/
var('x,y')
f=(x^3*y-x*y^3)/(x^2+y^2)
limit(limit(f,x=0),y=0)
0
var('x,y')
f=(x^3*y-x*y^3)/(x^2+y^2)
limit(limit(f,y=0),x=0)
0
Then is continuous on
.
var('x,y')
f(x,y)=(x^3*y-x*y^3)/(x^2+y^2)
f_xy=diff(diff(f,x),y)
fxy=limit(limit(f_xy,x=0),y=0)
print fxy
fyx=limit(limit(f_xy,y=0),x=0)
print fyx
-1, 1
Since two limits are different, is not define at
.
Actually on
except
.
var('x,y')
f(x,y)=(x^3*y-x*y^3)/(x^2+y^2)
f_xy=diff(diff(f,x),y)
f_yx=diff(diff(f,y),x)
print bool(f_xy==f_yx) # except the origin
True
Then it is easy to show that ,
,
,
exist. However
because
and
. ■
This example shows second order partial derivatives and
may be different, so the order of differentiation cannot be interchanged arbitrarily. The following theorem provides a sufficient condition for changing the order of differentiation.
THEOREM 2 Equality of Mixed Partial Derivatives
Suppose and it's derivatives
,
and
are defined in a domain containing a point
and all are continuous at
. Then
.
As a consequence, if and
both exist and are continuous, then
. If third partial derivatives exist and are continuous, then
is true as well. These results holds for all higher order partial derivatives.
EXAMPLE5
For , find the second order partial derivatives.
http://matrix.skku.ac.kr/cal-lab/m-Sec13-3-Exm-5.html
[CAS] http://sage.skku.edu/ 또는 https://sagecell.sagemath.org/
Solution. ,
,
,
,
,
.
var('x,y')
f=sin(x-y)+exp(-x*y)
fxx=diff(f,x,2)
fyy=diff(f,y,2)
fxy=diff(f,x,y)
fyx=diff(f,y,x)
print fxx
print fyy
print fxy
print fyx
Answer : y^2*e^(-x*y) - sin(x - y)
x^2*e^(-x*y) - sin(x - y)
x*y*e^(-x*y) - e^(-x*y) + sin(x - y)
x*y*e^(-x*y) - e^(-x*y) + sin(x – y) ■
EXAMPLE6
Show that the partial derivatives and
both exist, but the function
is not continuous at
.
Solution. Since the definition of partial derivatives, we have
.
However, is not continuous at
because the limit of
does not exists as follows;
letting along the line
,
. ■
13.3 EXERCISES (Partial Derivatives)
http://matrix.skku.ac.kr/Cal-Book/part2/CS-Sec-13-3-Sol.html
1. Find partial derivatives with respect to and
for the function
.
Solution.
[CAS] http://sage.skku.edu/ 또는 https://sagecell.sagemath.org/
var('x,y')
f=24*x*y-6*x^2*y
fx=diff(f,x);fy=diff(f,y)
print fx
print fy
Answer : -12*x*y+24*y, -6*x^2 + 24*x
2. Find partial derivatives with respect to and
for the function
.
Solution.
3. Find all second order partial derivatives of the function .
Solution.
4. Find all second order partial derivatives of the function .
Solution.
5. Take an example of the solution for the .
http://matrix.skku.ac.kr/cal-lab/cal-12-2-6.html
[CAS] http://sage.skku.edu/ 또는 https://sagecell.sagemath.org/
Solution.
var('x, y')
f(x, y)=x*y+x^3+y^3
solve([derivative(f,x,2)+derivative(f,y,2)==0], x, y)
([x == -y], [1])
Answer : .
6-8. Laplace’s equation A classical equation of mathematics is Laplace’s equation, which arises in both theory and applications. It governs ideal fluid flow, electrostatic potentials, and the steady-state distribution of heat in a conducting medium. In two dimensions, Laplace’s equation is
Show that the following functions are harmonic; that is, they satisfy Laplace’s equation.
6.
http://matrix.skku.ac.kr/cal-lab/m-Sec13-3-Exs-6.html
[CAS] http://sage.skku.edu/ 또는 https://sagecell.sagemath.org/
Solution.
var('x, y')
u(x, y)=x*(x^2-3*y^2)
u_xx=diff(u(x,y), x, 2)
u_yy=diff(u(x,y), y, 2)
bool(u_xx+u_yy==0)
True
7. for any real number
.
True
8.
[CAS] http://sage.skku.edu/ 또는 https://sagecell.sagemath.org/
var('x, y')
u(x,y)= arctan(y/(x-1)) - arctan(y/(x+1))
u_xx=diff(u(x,y), x, 2)
u_yy=diff(u(x,y), y, 2)
bool(u_xx+u_yy==0)
True
9. The volume of a right circular cone of radius and height
is
. Show that if the height remains constant while the radius changes the volume satisfies
.
10. Let . Show that
.
13.4 Differentiability and Total Differentials
Differentiability
Once again we try to generalize, the definition of differentiability of one variable function at
. Recall that
is differentiable at
, with its derivative
, if
. Using the
definition of limit, this means the following: Given any
,
for sufficiently small
. That is,
with the condition that as
. We can generalize this definition to a function
. In the sequel, we assume that
is defined on some open subset of
.
The analogous requirement with several variables is the definition of differentiability for functions or two (or more) variables.
DEFINITION1
The function is differentiable at
provided
and
exist and the change
equals
where for fixed and
,
and
are functions that depend only on
and
, with
as
. A function is differentiable on an open region
if it is differentiable at each point of
.
Several observations are needed here. First, the definition extends to functions of more than two variables. Second, we will show how differentiability is related to linear approximation and the existence of a tangent plane in Section 13.7. Finally, the conditions of the definition are generally difficult to verify. The following theorem may be useful in checking differentiablity.
THEOREM 2 Conditions for Differentiability
Suppose the function has partial derivatives
and
defined on an open region
and
and
are continuous on
. Then
is differentiable at every point of
.
This theorem states that existence of and
at
is not enough to ensure differentiability of
at
; we also need their continuity. Polynomials and rational functions are differentiable at all points of their domains, as are compositions of exponential, logarithmic and trigonometric functions with other differentiable functions.
We close with the analog of Theorem 3 in Section 3.1, which states that differentiability implies continuity.
THEOREM 3 Differentiability Implies Continuity
If a function is differentiable at
, then it is continuous at
.
Proof. By the definition of differentiability,
,
where as
. Because
is assumed to be differentiable, as
and
approach
, we see that
.
Also, because , it follows that
,
which implies continuity of at
. ■
EXAMPLE1
Discuss the differentiability and continuity of the function
Solution. As the definition of a rational function, is continuous and differentiable at all points
. The interesting behavior occurs at the origin. It can be shown that if the origin is approached along the line
, then
.
Therefore, the value of the limit depends on the direction of approach, which implies that the limit does not exist, and is not continuous at
. By Theorem 3,
is not differentiable at
.
Let’s look at the first partial derivatives of at
. A short calculation shows that
.
That is, the partial derivatives exist at .
Despite the fact that is not differentiable at
, its first partial derivatives exist at
. Existence of first partial derivatives at a point is not enough to ensure differentiability at that point. As expressed in Theorem 2, continuity of first partial derivatives is required for differentiability. It can be shown that
and
are not continuous at
. ■
Total Differential
The partial derivative introduced in the previous section tells us how a function changes when we vary one variable and hold the others fixed. Sometimes we wish to know how a function changes when more than one variable is changed. In section 3.4, we dealt with differential of a function at
. Recall that
.
In this section we wish to generalize this for the function of two variables. Let be a function of two variables. Assume that
has first order partial derivatives
and
at
in some neighborhood of
. Let us also assume for convenience that
and
are also continuous.
Let and
represent infinitesimal changes in
and
, respectively, at
. Let
be the increment of the function
. That is,
.
We add and subtract in
, and get
Using the mean value theorem in the interval , there exists
between
and
such that
.
Similarly using the mean value theorem in the interval , there exists
between
and
such that
.
Using and
, in
, we get
.
The above equation can be rewritten as follows: there exist ,
such that
.
Note that as andy
, we have
and
. Thus under the limiting case we have
.
The is called the differential or total differential of
. At time in infinitesimal case we write
and
, then the differential can be written as
.
In case, is a function of three variables then the total differential is given by
.
EXAMPLE2
If , find the total differential of
.
Solution. By definitions of the total differential, we have
.
The first order partial derivatives of with respect to
and
are as follows :
,
Hence, we obtain
. ■
EXAMPLE3
The base diameter and height of a cylinder are measured as and
, respectively, with a possible error in measurement of as much as
for each. Estimate maximum relative error and percentage error in the calculated volume of the cylinder.
Solution. The volume of a cylinder with base diameter
cm and height
cm is
. Taking natural logarithms on both sides of this equation, we get
.
Now we take the differentials on both sides to have
.
is a maximum when
and
are positive. Therefore, we take
and
along with
. This gives
.
The maximum relative error in the calculated volume is .
Thus the percentage error in the calculated volume is 5.25%. ■
EXAMPLE4
The dimensions of a rectangular cuboid are measured to be 70 cm, 60 cm and 50 cm, with a possible error in measurement as much 0.3 cm in each. Calculate differential and increment to estimate the maximum error in the volume of the rectangular cuboid.
Solution. Let ,
and
be the dimensions of the rectangular cuboid.
Then the volume is . By
, since
,
so
if ,
,
and
.
Next we will find the increment of as follows
. ■
Given , we have found the total differential
. If we have the total differential
, can we find
?
EXAMPLE5
If , find
.
Solution. By ,
.
Integrating with respect to
, we obtain
,
where is a constant with respect to
, that is a function of
. Next we differentiate both sides of
with respect to
, and we have
.
Then . Thus
.
Integrating with respect to
, we have
where is a constant. Putting this in
, we have
. ■
13.4 EXERCISES (Differentiability and Total Differential)
http://matrix.skku.ac.kr/Cal-Book/part2/CS-Sec-13-4-Sol.html
1. Find the total differential of when
.
Solution.
2. If , find the total differential of
.
Solution.
3. The base diameter and height of a right circular cone are measured as 10cm and 25cm, respectively, with a possible error in measurement of as much as for each. Estimate maximum relative error and percentage error in the calculated volume of the right circular cone.
Solution. Since the volume of right circular cone is ,
.
Let , then
.
The maximum relative error is and the percentage error is 3.2%.
4. Find an approximation using total differential.
(1) Find an approximation of when
.
(2) Find an approximation of .
Solution. (1) Since and
, the linear approximation is
Then .
(2) Let . Since
and
,
the linear approximation is
.
13.5 The Chain Rule
The Chain Rule
Let . If
and
depend on other variables, we would like to find derivatives or partial derivatives of
with respect to these independent variables. There are several variations that can occur. For example,
and
can be function of another variable, say
, or
and
can be functions of two variables
and
. The Chain Rule for multivariable function helps to find derivative and partial derivatives of compositions of functions. Let us consider the first case, where
and
are functions of
.
THEOREM 1 Chain Rule (One Independent Variable)
If is a function of
and
, and
and
are differentiable functions of
, then the composition
is a differentiable function of
and
.
Proof. If and
and
are both functions of the parameter of
, then
is the independent variable. Using
in Section 13.4, we have
.
When and
. By hypothesis
and
are continuous
or, . (See Figure 1.) ■
Figure 1
THEOREM 2
Let have continuous first order partial derivatives and
be a differentiable function of
. Then the composition
is a differentiable function of
and
.
Proof. Using in Section 13.4, we have
.
Taking the limit of the above equation as gives.
In particular
and
.
Taking the limit of the above equation as gives
or, . (See Figure 2.) ■
Figure 2
THEOREM 3 Chain Rule (Two Independent Variables)
Let have continuous first order partial derivatives with respect to
and
. Let
and
also have continuous first order partial derivatives with respect to
and
. Then
.
Proof. Using in Section 13.4, we have
for some . When
as
.
Now taking the limit of the above equation as gives
.
In other words,
.
It may be shown, similarly that
. (See Figure 3.)
■
Figure 3
THEOREM 4
Let have continuous first order partial derivatives with respect to
,
,
. Let
,
and
be functions of independent variables
and
. Also assume that
,
and
have continuous first order partial derivatives with respect to
and
. Then
and
. (See Figure 4.)
Figure 4
THEOREM 5
If and
then
and
. (See Figure 5.)
Figure 5
EXAMPLE1
If where
and
, find
.
Solution. .
Using the chain rule.
■
EXAMPLE2
If where
and
, find
.
Solution. Let us look at the tree diagram in Figure 6.
Figure 6
Using the chain rule gives
Now
.
.
Therefore
.
http://matrix.skku.ac.kr/cal-lab/cal-12-2-Exm6.html
[CAS] http://sage.skku.edu/ 또는 https://sagecell.sagemath.org/
var('u, x, y, z')
u=ln(x^2+y^2+z^2)
u
log(x^2 + y^2 + z^2)
dudx=diff(u, x).factor()
dudx
2*x/(x^2 + y^2 + z^2)
dudy=diff(u, y).factor()
dudy
2*y/(x^2 + y^2 + z^2)
dudz=diff(u, z).factor()
dudz
2*z/(x^2 + y^2 + z^2)
y=x*sin(x)
dydx=diff(y, x).factor()
dydx
x*cos(x) + sin(x)
z=x*cos(x)
dzdx=diff(z, x).factor()
dzdx
-x*sin(x) + cos(x)
(dudx+dudy*dydx+dudz*dzdx).subs(y=x*sin(x), z=x*cos(x)).trig_simplify()
Answer : 2/x ■
EXAMPLE3
If where
, find
.
http://matrix.skku.ac.kr/cal-lab/m-Sec13-5-Exm-3.html
Solution. Using the chain rule, we have
.
Since ,
,
.
Therefore, we have
,
,
Since ,
.
Therefore
. ■
EXAMPLE4
If where
and
, find
,
,
and
.
Solution. Taking the first order partial derivatives with respect to and
of
and
, we obtain
Solving (1) and (2), we have
and
.
Solving (3) and (4), we have
and
.
To find , first we can get the following
.
Thus we have
.
Therefore .
http://matrix.skku.ac.kr/cal-lab/m-cal-13-5-Exm-4.html
[CAS] http://sage.skku.edu/ 또는 https://sagecell.sagemath.org/
var('r,t,x,y,r_x,t_x,r_y,t_y')
z=r^2*exp(-t)
x=r*cos(t);y=r*sin(t)
solve([1==cos(t)*r_x-r*sin(t)*t_x,0==sin(t)*r_x+r*cos(t)*t_x],r_x,t_x)
r_x = cos(t)/(sin(t)^2 + cos(t)^2), t_x =-sin(t)/(r*sin(t)^2 +r*cos(t)^2)]]
r_x=cos(t)/(sin(t)^2 + cos(t)^2)
t_x=-sin(t)/(r*sin(t)^2+r*cos(t)^2)
print r_x.simplify_trig()
print t_x.simplify_trig()
cos(t), -sin(t)/r
solve([0==cos(t)*r_y-r*sin(t)*t_y,1==sin(t)*r_y+r*cos(t)*t_y],r_y,t_y)
r_y = sin(t)/(sin(t)^2 + cos(t)^2), t_y = cos(t)/(r*sin(t)^2 +r*cos(t)^2)
r_y = sin(t)/(sin(t)^2 + cos(t)^2)
t_y = cos(t)/(r*sin(t)^2+r*cos(t)^2)
print r_y.simplify_trig()
print t_y.simplify_trig()
sin(t), cos(t)/r
z_y=diff(z,r)*r_y+diff(z,t)*t_y
z_y.simplify_trig()
(2*r*sin(t) - r*cos(t))*e^(-t)
z_yx=diff(z_y,r)*r_x+diff(z_y,t)*t_x
z_yx.simplify_trig()
-(sin(t)*cos(t) + 2*cos(t)^2 – 1)*e^(-t). ■
Implicit Differentiation
http://archives.math.utk.edu/visual.calculus/3/implicit.7/index.html
(Ⅰ) For the relationship between two variables and
, we say that
is an implicit function of
if we are given an equation
Here can be thought of as a single-valued function in a small neighborhood of
. If the relationship where
exists, then
is an explicit function of
. The following theorem holds for implicit function
, if
has continuous first order partial derivatives of
with respect to
in the neighborhood at the point where
.
THEOREM 6 Implicit Differentiation
Let have continuous first order derivatives on its domain. Suppose that
defines
as a differentiable function of
. Then
, provided
.
Figure 7
Proof. We can apply the chain rule to differentiate both sides of the equation with respect to
. Since both
and
are functions of
, we get
.
Then, if
. ■
Now we suppose that
is given implicity as a function
by an equation of the form . This means that
for all
in the domain of
. If
and
are differentiable, then we can apply Chain Rule to differentiate the equation
as follows:
THEOREM 7 Implicit Differentiation
Let be differentiable on its domain and suppose that
defines
as an differentiable function of
and
. Then
, where
.(See Figure 8.)
Figure 8
Proof. We assumed that defines
implicitly as a function of
and
. Then
.
So this equation becomes
because
and
.
Similarly,
because
and
.
If , then we have
. ■
,
we obtain the derivative with respect to
,
by solving the system of equations above, we find . Let us assume that the determinant
,
then
,
.
EXAMPLE5
If , find
.
Solution. Let . Then
.
Therefore, from formula , we obtain
.
var('x,y')
f=x*sin(y)+y*cos(x)
f_x=diff(f,x);f_y=diff(f,y)
dydx=-f_x/f_y
dydx
Answer : (y*sin(x) - sin(y))/(x*cos(y) + cos(x))
EXAMPLE6
Suppose is an implicit function of
and
on any rectangular region that satisfy
. Find
and
.
Solution. Let . Since
,
and ,
from , we obtain
. ■
var('x,y,z')
f=x^3*exp(y+z)-y*sin(x-z)
f_x=diff(f,x);f_y=diff(f,y);f_z=diff(f,z)
dzdx=-f_x/f_z;dzdy=-f_y/f_z
print dzdx
print dzdy
Answer : -(3*x^2*e^(y+z)-y*cos(x-z))/(x^3*e^(y+z)+y*cos(x-z))
-(x^3*e^(y+z)-sin(x-z))/(x^3*e^(y+z)+y*cos(x-z))
EXAMPLE7
If equation , find
and
.
Solution. Let . Then
,
and
.
Therefore, from formula , we obtain
and
. ■
13.5 EXERCISES (The Chain Rule)
http://matrix.skku.ac.kr/Cal-Book/part2/CS-Sec-13-5-Sol.html
1. If where
,
and
, find
and
at
and
.
Solution.
.
2. Find and
if
, where
.
Solution. (1) ,
.
(2)
.
3. Find of
.
Solution.
4. Find of
.
Solution. Take a partial derivative of both sides with respect to , then we have
.
.
5. Find and
when
is an implicitly defined function of
and
in
.
Solution. If
,
,
6. Find ,
of
.
Solution.
because
.
because
.
7. Find when
.
Solution.
8. Let with
.
(a) Show that and
exist everywhere.
(b) Are and
continuous at the origin? Justify your answer.
http://matrix.skku.ac.kr/cal-lab/cal-12-2-5.html
[CAS] http://sage.skku.edu/ 또는 https://sagecell.sagemath.org/
Solution. (a) If ,
.
var('x, y, h')
f(x, y)=x*y/(x^2+y^2)
fx(x, y)=limit((f(x+h, y)-f(x, y))/h, h=0)
fy(x, y)=limit((f(x, y+h)-f(x, y))/h, h=0)
print(fx(x, y))
print(fy(x, y))
-(x^2*y - y^3)/(x^4 + 2*x^2*y^2 + y^4)
(x^3 - x*y^2)/(x^4 + 2*x^2*y^2 + y^4)
Thus and
exist everywhere.
(b)
print(limit(fx(x, 0), x=0))
print(limit(fx(0, y), y=0))
0
Infinity
is not continuous at the origin.
print(limit(fy(x, 0), x=0))
print(limit(fy(0, y), y=0))
Infinity
0
is not continuous at the origin.
9. A function is called a homogeneous function of degree
if all the terms in
are of degree
. In other words,
for any parameter
. If
is a homogeneous function of degree n then show that
This is also called Euler’s theorem for homogeneous function.
10. Verify the Euler’s theorem for the following:
(i) .
(ii) .
(iii) .
(iv) .
(v) .
(vi) .
11. If is a homogeneous function of degree
in
,
and
then show that
.
12.Let . Find
at
,
.
13. Let is a homogeneous function of degree
in
and
. If
then show that
.
14.Let . Show that
.
15.Let . Show that
.
16. Show that is a solution of
for all
and
assuming that
is a constant.
17. If is the solution of the equation
with the condition that
as
, find the values of
and
.
18. Let ,
,
and
. Find
.
19. Let ,
,
. Find
at
.
13.6 Directional Derivatives and Gradient
The partial derivatives and
are the rates of change of
at
in the positive
- and
-directions. Rates of change in other directions are given by directional derivatives. We open this section by defining directional derivatives and then use the Chain Rule to derive a formula for their values in terms of
- and
-derivatives. Then we study gradient vectors and show how they are used to determine how directional derivatives at a point change as the direction changes, and, in particular, how they can be used to find the maximum and minimum directional derivatives at a point.
In this section we introduce a type of derivative, called a directional derivative, that enables us to find the rate of change of a function of two or more variables in any direction.
Directional Derivatives
DEFINITION1 Directional Derivatives
Let be a unit vector. The directional derivative of
at
in the direction of
is defined as
if the limit exist.
1. Note that if , then
and if
, then
.
2. Consider a curve in the domain of
. Note that
is the line passing through
in the direction of
furthermore,
and
. Then it is easy to see that
.
EXAMPLE1
Find the directional derivative of at
in direction of
.
http://matrix.skku.ac.kr/cal-lab/m-Sec13-6-Exm-1.html
Solution.
var('x,y,h')
f(x, y)=4*x^2*y+3*x*y+6*x+2
limit((f(1+h,-1+h*0) - f(1,-1))/h, h=0)
Answer : -5 ■
THEOREM 2
Suppose that is differentiable at
and
is any unit vector. Then
Proof.
(by the Chain rule)
. ■
EXAMPLE2
Find the directional derivative if
and is the unit vector given by angle
.
http://matrix.skku.ac.kr/cal-lab/m-Sec13-6-Exm-2.html
[CAS] http://sage.skku.edu/ 또는 https://sagecell.sagemath.org/
Solution. When , the unit vector is
.
Since and
,
and
.
Then . ■
var('x,y')
f(x,y)=x^3-3*x*y+4*y^2
delf=vector([diff(f(x,y),x),diff(f(x,y),y)])
delfp=delf.subs(x=1,y=2)
u=vector([cos(pi/6),sin(pi/6)])
dirdr=delfp.dot_product(u)
print dirdr
Answer : -3/2*sqrt(3) + 13/2
Gradient Vectors
For convenience, we define the gradient of a function to be the vector-valued function whose components are the first-order partial derivatives of , as specified in Definition 3. We denote the gradient of a function
by grad
or
.
DEFINITION3
The gradient of is the vector-valued function
,
provided both first order partial derivatives exist.
3. In view of Theorem 2, we have
.
EXAMPLE3
Find the gradients of
(i) ,
(ii) .
Solution.
(i) http://matrix.skku.ac.kr/cal-lab/m-Sec13-6-Exm-3.html
[CAS] http://sage.skku.edu/ 또는 https://sagecell.sagemath.org/
var('x,y')
f(x, y)=4*x^2+3*y^2-5*x*y
f.gradient()
Answer : (x, y) |--> (8*x - 5*y, -5*x + 6*y)
(ii)
var('x,y')
f(x, y)=x*y/(x^2+y^2+1)
f.gradient()
Answer : (x, y) |--> (-2*x^2*y/(x^2 + y^2 + 1)^2 + y/(x^2 + y^2 + 1), -2*x*y^2/(x^2 + y^2 + 1)^2 + x/(x^2 + y^2 + 1))
. ■
Interpretations of the Gradient
The gradient is not limited to calculating partial derivatives; it plays important roles in multivariable calculus. Our present goal is to develop some intuition about the meaning of the gradient.
From Remark 3, . Using properties of the dot product, we have
where is the angle between
and
. It follows that
has its maximum value when
, which corresponds to
. Therefore,
has its maximum value and
has its greatest rate of increase when
and
point in the same direction. Notice that when
, the actual rate of increase is
.
Similarly, when , we have
, and
has its greatest rate of decrease when
and
point in opposite directions. The actual rate of decrease is
. These observations are summarized as follows:
Write these as three observations:
1. The gradient points in the direction of steepest ascent at
.
2. The negative of the gradient points is the direction of steepest descent.
3. Notice that when the angle between
and
is
, which means
and
are orthogonal.
EXAMPLE4
(a)Find the derivative of at the point
in the direction of the unit vector
.
(b) What is the direction in which the function increase most rapidly at
in
-plane region?
(c) Determine the direction of zero change in at (3, 4).
Solution. (a) Since ,
,
hen the directional derivative is
.
(b) The function increases most rapidly in the direction of the gradient. The gradient is
.
Its direction is
.
(c) The directions of zero change in at (3, 4) are the directions orthogonal to
.
and
.
Computing shows that they are perpendicular.
. ■
The Gradient and Level Curves
Note that Observation 3 says that in the direction orthogonal to the gradient , the function
does not change at
. Recall the curve
, where
is a constant, is a level curve, on which function values is constant. Combing these two observations, we conclude that gradient
is orthogonal to the line tangent to the level curve through
.
THEOREM 4 The Gradient and Level Curves
Given a function differentiable at
, the line tangent to the level curve of
at
is orthogonal to the gradient
.
Proof. A level curve of the function is a curve in the
-plane of the form
, where
is a constant. By Theorem 6 in section 13.5, the slope of the line tangent to the level curve is
.
It follows that any vector that points in the direction of the tangent line at the point is a scalar multiple of the vector
.
At that same point, the gradient points in the direction
.
The dot product of and
is
which implies that and
are orthogonal to each other. ■
An immediate consequence of Theorem 4 is an alternative equation of the tangent line. The curve described by can be viewed as a level curve for a surface. By Theorem 4, the line tangent to the curve at
is orthogonal to
. Therefore, if
is a point on the tangent line, then
, which, when simplified, gives an equation of the line tangent to the curve
.
EXAMPLE5
Consider the upper sheet of a hyperboloid of two sheets.
(a)Verify that the gradient at is orthogonal to the corresponding level curve at that point.
(b) Find an equation of the tangent line to the level curve at .
Solution. (a) You can verify that is on the surface; therefore,
is on the level curve corresponding to
. Setting
in the equation of the surface and squaring both sides, the equation of the level curve is
or
, which is the equation of an ellipse. Differentiating
with respect to
gives
, which implies that the slope of the level curve is
. Therefore, at the point
, the slope of the tangent line is
. Any vector proportional to
has slope
and points in the direction of the tangent line.
We now compute the gradient:
.
It follows that . The tangent vector
and the gradient are orthogonal because
.
(b) An equation of the line tangent to the level curve at is
.
Then the tangent line is . ■
Functions of Three Variables
Let be a function of three variables defined on some domain
in
. The notion of directional derivative and gradient of
can be defined just by introducing extra component z in definition of two variable function
. For the sake of completion, let us write these definitions:
DEFINITION5
Let be a function of three variables defined in a domain
. Let
and
be an unit vector in
. Then the directional derivative of
at
in the direction of
is given by
if the right hand side limit exists.
Again using chain rule it turns out that
.
DEFINITION6
Let be a function of three variables defined in a domain
. Let
and
possesses continuous first order partial derivative at
. Then the gradient of
at
is defined as
EXAMPLE6
Find the directional derivative of the function at
in direction of
.
Solution. We have ,
, and
so that
Since and
is a unit vector in the indicated direction. From last definition we obtain
. ■
Suppose has continuous first order partial derivatives. Then the surface
is one of the level surfaces of . Let
be differentiable curves passing through the point on
where
,
and
. For all
, we obtain
.
Then using the chain rule, we have
.
.
Therefore
,
where is a tangent vector of the curve. What we conclude from this is that the gradient passing through
is perpendicular to the tangent vector of all differentiable curves along
. (See Figure 1.) Thus
is perpendicular to the tangent vector of all differentiable curves passing through
on
. Therefore, these lines are perpendicular to
and in the plane passing through
.
Figure 1 is perpendicular to tangent vector of differentiable function at
. So tangent vector is on the surface.
EXAMPLE7
Find the directional derivative of the function at the point
in the direction of the vector
.
Solution. The unit vector of is
with .
The partial derivatives of at
are
.
The gradient of at
is
.
Hence the directional derivative of at
in the direction of
is
. ■
EXAMPLE8
Estimate how much the value of will change if the point
moves
unit from
straight toward
.
Solution. We first find the directional derivative of at
in the direction of the vector
. The unit vector of
is
.
The gradient of at
is
.
Therefore,
.
The change in
that results from moving
unit away from
in the direction of
is approximately
unit. ■
EXAMPLE9
Plot the contour plot and level surface of
.
http://matrix.skku.ac.kr/cal-lab/cal-DirectionalDerivative.html
[CAS] http://sage.skku.edu/ 또는 https://sagecell.sagemath.org/
html("<i> <b> Directional derivative 시각화 <p></p> </b>" )
var('x,y,t,z')
f(x,y)=2*sin(x)*cos(y)
line_thickness=3
surface_color='blue'
plane_color='purple'
line_color='red'
tangent_color='green'
gradient_color='orange'
@interact
def myfun(location=input_grid(1, 2, default=[0,0], label = "Location (x,y)", width=2), angle=slider(0,2*pi, label = "Angle"),
show_surface=("Show surface", True), show_contour=("Show Contour Plot", True)):
location3d = vector(location[0]+[0])
location = location3d[0:2]
direction3d = vector(RDF, [cos(angle), sin(angle), 0])
direction=direction3d[0:2]
cos_angle = math.cos(angle)
sin_angle = math.sin(angle)
df = f.gradient()
direction_vector=line3d([location3d, location3d+direction3d], arrow_head=True, rgbcolor=line_color, thickness=line_thickness)
curve_point = (location+t*direction).list()
curve = parametric_plot(curve_point+[f(*curve_point)], (t,-3,3),color=line_color,thickness=line_thickness)
plane = parametric_plot((cos_angle*x+location[0],sin_angle*x+location[1],t), (x, -3,3), (t,-3,3),opacity=0.8, color=plane_color)
pt = point3d(location3d.list(),color='green', size=10)
tangent_line = parametric_plot((location[0]+t*cos_angle, location[1]+t*sin_angle, f(*location)+t*df(*location)*(direction)), (t, -3,3), thickness=line_thickness, color=tangent_color)
picture3d = direction_vector+curve+plane+pt+tangent_line
picture2d = contour_plot(f(x,y), (x,-3,3),(y,-3,3), plot_points=100)
picture2d += arrow(location.list(), (location+direction).list(),color="red")
picture2d += point(location.list(),rgbcolor='green',pointsize=40)
if show_surface:
picture3d += plot3d(f(x,y), (x,-3,3),(y,-3,3),opacity=0.7)
dff = df(location[0], location[1])
dff3d = vector(RDF,dff.list()+[0])
picture3d += line3d([location3d, location3d+dff3d], arrow_head=True, rgbcolor=gradient_color, thickness=line_thickness)
picture2d += arrow(location.list(), (location+dff).list(), rgbcolor=gradient_color, width=line_thickness)
if show_contour:
show(picture2d, aspect_ratio=1)
show(picture3d,aspect=[1,1,1], axes=True)
Figure 2
Figure 3
■
13.6 EXERCISES (Directional Derivatives and Gradient)
http://matrix.skku.ac.kr/Cal-Book/part2/CS-Sec-13-6-Sol.html
1. Find the directional derivative of the function at the point
in the direction of the vector
.
Solution. ,
⇒ ,
.
2. Find the directional derivative of the function at the point
in the direction of the vector
.
Solution. =>
and
.
,
⇒
,
.
3. Find the directional derivative of the function at the point
in the direction of the vector
.
Solution.
.
4. If , find
.
http://matrix.skku.ac.kr/cal-lab/cal-12-2-2.html
[CAS] http://sage.skku.edu/ 또는 https://sagecell.sagemath.org/
Solution.
var('x, y, z')
f(x, y, z)=1/sqrt(x^2+y^2+z^2)
f.gradient() # Find gradient
(x, y, z) |--> (-x/(x^2 + y^2 + z^2)^(3/2), -y/(x^2 + y^2 + z^2)^(3/2), -z/(x^2 + y^2 + z^2)^(3/2))
Therefore,
.
5. Find the gradient of at the point
.
Solution.
.
6. Use the definition of the gradient, assume that and
are differentiable function on
, and let
be a constant. Prove the following gradient rules.
(1)
(2)
(3)
(4)
Solution. and
are differentiable function on
(1)
.
(2)
.
(3)
(4)
.
7. Find the gradient of .
Solution. ,
.
13.7 Tangent Plane
In this section, we introduce tangent plane to a surface and look at how to find its equations. Tangent plane is extension of tangent line to a one variable function
.
Recall that if be a differentiable function of one variable, then the equation of the tangent line at
is given by
.
Tangent Plane
A tangent plane is defined to be a plane that is just tangent to a surface (it touches the surface in only one point).
To derive the equation of tangent plane, we will use Theorem 4 in section 13.6: tangent plane is orthogonal to the gradient. Let be a differentiable function in its domain. If
is surface which is a level surface of
and
is a point on
, then we have proved that the gradient
is orthogonal to the tangent line to level curves. (See Figure 1.)
Figure 1 Tangent Plane to a surface
Thus one can define tangent plane at to
as a plane which is perpendicular to
, provided
.
Thus, if and
are points on the tangent plane and
and
are their corresponding position vectors, respectively, a vector equation of tangent plane is
where .
The definition of tangent plane is given below:
DEFINITION1 Tangent Plane
Let be a point on the level surface
where
is not
. Then an equation of the tangent plane at
is
EXAMPLE1
Find an equation of the tangent plane to the graph of the sphere at
.
Solution. By defining , we find that the given sphere is the level surface
passing through
. Now,
,
,
so that
and
.
It follows the form in Definition 1 that an equation of the tangent plane is
or
.
See in Figure 2.
Figure 2 Tangent Plane in Example 2
■
The equation of the tangent plane obtained in for a function
at a point
where
can be obtained using Definition 1 the graph of the function
can be thought of level surface of
, where
. In this case we have
,
and
.
Hence by Definition 1, the equation of tangent plane is
.
This implies that the equation of tangent plane is
.
DEFINITION2 Equation of Tangent Plane
Let be a function of two variables having first order continuous partial derivatives in some domain. Let
be a point in the domain of
. Then equation of tangent plane to the surface
at the point
is given by
.
EXAMPLE2
Find the tangent plane to the surface at the point
. (See figure 3.)
Figure 3
Solution. Let . Then
and
. Since
and
, the equation of tangent plane is
.
[CAS] http://sage.skku.edu/ 또는 https://sagecell.sagemath.org/
var('x,y')
f(x,y)=9-x^2-y^2
a,b=1,2
fx=f.diff(x)(a,b)
fy=f.diff(y)(a,b)
T(x,y)=f(a,b)+fx*(x-a)+fy*(y-b)
show(T(x,y))
Answer :
p=plot3d(f,(x,-2,2),(y,-3,3),color='red',opacity=0.3);
pt=point3d((a,b,f(a,b)),pointsize=50)
tgt=plot3d(T,(x,a-0.6,a+0.6),(y,b-0.6,b+0.6),color='blue',opacity=0.5);
p+pt+tgt
■
Normal Line
Let be a function having continuous first order partial derivatives. Then the gradient to the surface
at
is given by
where .
We know that the gradient is perpendicular to the tangent plane to the surface at the point . Hence the parametric equations of the normal line is given by
,
,
In case and
, the symmetric equation of the normal line to the surface at
can be written as
.
EXAMPLE3
Find the tangent plane and the normal line at the point to the surface
. (See Figure 4.)
Figure 4 A tangent plane and a normal vector to the surface
at
.
Solution. 1. The partial derivatives of at
are
By Definition 1, the equation of a tangent plane at is
or
.
By , the equation of normal line is
(
). ■
EXAMPLE4
Find the tangent plane and the normal line at the point to the surface
.
Solution. Given equation can be written as
Then the equation can be considered as a level surface of
.
Therefore our tangent plane at is the plane perpendicular to the gradient
at
. Now we have
,
,
.
The equation of tangent plane is
or .
The equation of normal line is
http://matrix.skku.ac.kr/cal-lab/cal-12-3-Exm7.html
var('x,y,z,t')
F=ln(x^2+y^2)-z
g= F.gradient()
normal=g.subs(x=1,y=2,z=ln(5))
PP_0=vector([x-1,y-2,z-ln(5)])
T=normal.dot_product(PP_0)
Nline=vector([1+normal[0]*t,2+normal[1]*t,ln(5)+normal[2]*t])
print T==0
print Nline
■
EXAMPLE5
Use sage to find the tangent plane and the normal line to the surface at
,
. Also plot all these together.
http://matrix.skku.ac.kr/cal-lab/m-Sec13-7-Exm-5.html
Solution.
[CAS] http://sage.skku.edu/ 또는 https://sagecell.sagemath.org/
var('x,y,z,t')
f(x,y)=4*x*y*exp(-x^2-y^2)
F(x,y,z)= z-4*x*y*exp(-x^2-y^2)
a,b=1,0.5
c=f(a,b)
delF=F.gradient()
delFp=delF(a,b,c)
tangent_plane=delFp.dot_product(vector([x-a,y-b,z-c]))==0
print "The tangent plane is"
show(tangent_plane)
fx=f.diff(x)(a,b)
fy=f.diff(y)(a,b)
normal_line= (x==a+t*fx, y==b+t*fy, z==c-t)
print "The normal line is"
show(normal_line)
Answer : The equation of the Tangent Plane is:
0.573009593720380x−0.573009593720380y+z−0.859514390580570=0
The equation of the normal line is:
(x=−0.573009593720380t+1,y=0.573009593720380t +0.500000000000000, z=−t+0.573009593720380)
Figure 5
pt=point3d((a,b,f(a,b)), pointsize=50, color='red')
p=plot3d(f, (x,-2,2),(y,-2,2), color='green', opacity=0.2)
tangent=implicit_plot3d(delFp.dot_product(vector([x-a,y-b,z-c]))==0,(x,a-0.3,a+0.3),(y,b-0.3,b+0.3),(z,c-0.3,c+0.3), color='blue', opacity=0.5)
normal=parametric_plot3d(vector([a+t*fx,b+t*fy, c-t]),(t,-1,1),color='red')
p+pt+tangent+normal
Answer : [[x == 0, y == 0], [x == -1/2*sqrt(2), y == -1/2*sqrt(2)], [x ==1/2*sqrt(2), y == -1/2*sqrt(2)], [x == -1/2*sqrt(2), y == 1/2*sqrt(2)], [x == 1/2*sqrt(2), y == 1/2*sqrt(2)]] ■
13.7 EXERCISES (Tangent Plane)
http://matrix.skku.ac.kr/Cal-Book/part2/CS-Sec-13-7-Sol.html
1. Find the equation of the tangent plane at the point to the surface
.
Solution.
⇒ The equation of the tangent plane is
.
Answer : .
2. Find the equation of the tangent plane and the normal line at the point to the surface
.
Solution.
⇒ and
Hence at the point , the tangent plane is
and the normal line is
(that is,
).
3. Find so that all tangent planes to the surface
pass through the origin
for all
.
Solution. Let . Then we have that
and
.
Since every tangent plane intersects the origin, the equation of the tangent plane at a point on the surface should satisfies
.
We then have
.
Simplifying the equation, we obtain
,
which implies .
4. Find the tangent plane of the surface at
.
Solution. Let .
Due to implicit differentiation, we have
,
and
.
In particular, at we have
.
Thus, the tangent plane is . Simplifying it, we have
.
5. Find the equation of the tangent plane and the normal line to the surface at
.
Solution.
Equation of tangent plane:
.
or
.
Equation of normal line:
,
.
6. Let be the surface whose equation in cylindrical coordinates is
. Find the tangent plane and the normal line to
at the point
in rectangular coordinates.
Solution. The equation of in rectangular coordinates is given by
.
If we regard as a level surface of
at , then the normal vector of the tangent plane is
, where
and hence
Therefore, the equations of the tangent plane is
or
.
And the normal line is
.
7. Let if
and let
In the direction of what unit vectors does the directional derivative of
at
exist?
Solution. Suppose we wish to compute the directional derivative of in the direction of a unit vector
Then
and the limit does not exist unless or
Hence the directional derivative of
at
exists in the directions of
and
or their unit scalar multiples.
8-9. Find the tangent plane to the given surface at the indicated point.
8. at
.
http://matrix.skku.ac.kr/cal-lab/cal-12-3-4.html
[CAS] http://sage.skku.edu/ 또는 https://sagecell.sagemath.org/
Solution.
var('x, y, z, w')
f(x, y, z, w)=x^2+y^2+z^2-w
grad_f=f.gradient()
fx, fy, fz, fw=grad_f
fx(-1, 0, 0, 1)*(x+1)+fy(-1, 0, 0, 1)*(y-0)+fz(-1, 0, 0, 1)*(z-0)+fw(-1, 0, 0, 1)*(w-1)==0
Answer : -w - 2*x - 1 == 0
9. at
.
http://matrix.skku.ac.kr/cal-lab/cal-12-3-11.html
Solution.
var('x, y, z')
f(x, y, z)=exp(3*y)*cos(2*x)-z
grad_f=f.gradient()
fx, fy, fz=grad_f
p1=implicit_plot3d(f(x, y, z)==0, (x, 0, pi/2), (y, -1, 1), (z, -1, 0), opacity=0.6)
p2=implicit_plot3d(fx(pi/3, 0, -1/2)*(x-pi/3)+fy(pi/3, 0, -1/2)*(y-0)+fz(pi/3, 0, -1/2)*(z+1/2)==0, (x, 0, pi/2), (y, -1, 1), (z, -1, 0), color='orange', opacity=0.6)
p3=point3d([pi/3, 0, -1/2], color='red')
p1+p2+p3
1/3*(pi - 3*x)*sqrt(3) - 3/2*y - z - 1/2 == 0
Answer :
10. Find the equations of the tangent plane and normal line at the point to the paraboloid
.
http://matrix.skku.ac.kr/cal-lab/m-Sec13-7-Exs-10.html
Solution.
[CAS] http://sage.skku.edu/ 또는 https://sagecell.sagemath.org/
var('x,y,z,t')
F=-x^2-y^2-z
g= F.gradient()
normal=g.subs(x=-1,y=1,z=-2)
PP_0=vector([x+1,y-1,z+2])
T=normal.dot_product(PP_0)
Nline=vector([-1+normal[0]*t,1+normal[1]*t,-2+normal[2]*t])
print T==0
print Nline
Answer : 2*x - 2*y - z + 2 == 0
(2*t - 1, -2*t + 1, -t – 2)
p1=implicit_plot3d(z==-x^2-y^2,(x,-7,7),(y,-7,7),(z,-7,7), opacity=0.2, color="red", mesh=True);
p2=implicit_plot3d(dF_p*(w-p)==0,(x,-7,7),(y,-7,7),(z,-7,7), opacity=0.2, color="blue",
mesh=True);
p3=parametric_plot3d((-2*t-1,2*t+1,t-2),(t,-1,1), opacity=1, color="red", mesh=True);
show(p1+p2+p3, aspect_ratio=1)
So tangency normal vector is clear.
11. Find the equations of the tangent plane and the normal line at the point to the surface
.
Solution. We have ,
,
.
Hence, .
Therefore by the equation of the tangent plane to the surface
at the point
is
.
Or, .
By the equation of the normal line at the point
to the surface
is
,
.
12. The surface and
meet in an ellipse
. Find parametric equations for the line tangent to
at the point
.
Solution. Let be a point on the intersection of these surfaces. Then the tangent line at
to the curve
is orthogonal to
and
. In particular, it is parallel to
.
The components of and the coordinates of
give us equations for the line. We have
,
.
Therefore .
Thus the tangent to the curve at
is parallel to the vector
. Therefore, its equation is
.
13. Sketch a level curve (ellipse) of passing through the pont
. Find a vector perpendicular to this ellipse at the point
.
Solution. The value of at the point
is
.
Therefore, the level curve of passing through
is
, which is an ellipse. Vector perpendicular to this ellipse at the point
is
.
13.8 Extrema of Multivariate Functions
We have seen how to find extreme values of a single variable function as an application to derivatives. In this section we look at how to find extrema of multivatiate functions as an application to partial derivatives. As with single variable function, concepts developed in this section are useful for practical optimization problems.
Local and Global Maximum and Minimum
We begin with the definition of local and relative extrema for a function of two variables and
.
DEFINITION1 Local Maximum and Minimum
(i) A point is a local maximum of a function
if
for all
near
in the domain of
.
(ii) A point is a local minimum of a function
if
for all
near
in the domain of
.
Let and suppose
has a local minimum at
. This means there exists positive real numbers
such that
for all
and
. Similarly, we can define a local maximum at
.
When in
,
is a single variable function. Suppose the first order partial derivatives of
exist at the point
, then we have
since it has the extreme value at
.
Likewise, when ,
has the extreme value at
and
. Therefore, if
is a local maximum or local minimum of
, then
This is in fact, a necessary condition for a point to be a point of local maximum/minimum of
. All such points where first order partial derivatives vanish are called critical points of
.
Figure 1
DEFINITION2
A critical point of a function is a point
in the domain of
for which
and
, or where one of its partial derivatives does not exist.
THEOREM 3 Local Extrema
If a function has a local extremum at point
and the first derivatives exist at this point, then
and
.
Theorem 3 says that if has a local maximum or local minimum at
, then
is a critical point of
. In general,
can be a critical point of
even though
is not a local maximum or local minimum value.
Figure 2
The discussion below/above suggests the next theorems.
Let us look at . The graph of
is shown in the Figure 1.
The point in Figure 2, is a critical point which is neither a local maximum nor local minimum. It is easy to see that
and
implies that
and
. That means
is the only critical point of
. However from the graph, it is clear that
is not a local maximum/local minimum. In fact if we take any small neighborhood around
, there are points
and
in this neighborhood where
and
. Because this surface looks like a saddle horse, such a critical point is called a saddle point.
Another way of looking at Theorem 3 is the following. Suppose has continuous first order partial derivatives. We know that
is the direction in which function increases with maximum speed. Thus, if
is a point of local maximum, then at
on the surface,
, there is no direction in which function can increase. Hence
which implies
. Similarly, at the point of local minimum
.
EXAMPLE1
Find all the critical points for .
Solution. The first partial derivatives are
and
.
Hence, and
implies
and
.
Thus the critical point is . ■
Figure 3
EXAMPLE2
Find all the critical points for .
Solution. The first partial derivatives are
and
.
Hence, and
implies
and
.
Thus, the critical point is . ■
EXAMPLE3
Find critical points of . Plot the surface and its contour lines.
http://matrix.skku.ac.kr/cal-lab/m-Sec13-8-Exm-3.html
[CAS] http://sage.skku.edu/ 또는 https://sagecell.sagemath.org/
Solution.
var('x,y')
f(x,y)=4*x*y*exp(-x^2-y^2)
fx=f.diff(x)
fy=f.diff(y)
sol=solve([fx==0,fy==0],[x,y],solution_dict=True)
sol
Answer : [{y: 0, x: 0}, {y: -1/2*sqrt(2), x: -1/2*sqrt(2)},
{y: -1/2*sqrt(2), x:1/2*sqrt(2)}, {y: 1/2*sqrt(2), x: -1/2*sqrt(2)},
{y: 1/2*sqrt(2), x:1/2*sqrt(2)}]
Here has five critical points
and
. This is clear from the graph of the surface
. Look at Figure 4.
Figure 4 Figure 5
It is also clear from the Figure Give Reference that f has two points of local maximum and two points of local minimum and a saddle point. Look at contour line of the function . Look at Figure 4.
It is clear from the contour lines of (See Figure 5.) that, near the point of local maximum and local minimum, level curves resembles concentric circles. Near saddle points, the level curves are hyperbolic looking shapes. ■
Second Derivative Test
We know that critical points are likely candidates for point of local maximum, local minimum or saddle point. Analogous to second derivative test for function of one variable, there is second derivative test for functions of two variables to classify critical points as local maximum, local minimum or saddle points.
The following theorem tells us how to decide the local maximum and local minimum of a given function at a critical point.
THEOREM 4 Second Derivatives Test
Suppose and
has continuous second-order partial derivatives in a neighborhood of
. Let
.
(1) If and
, then
is a local maximum.
(2) If and
, then
is a local minimum.
(3) If , then
does not have a local maximum or
minimum at . The point
is a saddle point.
In case (c) the point is called a saddle point of
and the graph of
crosses its tangent plane at
.
If , the test gives no information:
could have a local maximum or local minimum at
, or
could be a saddle point of
.
To remember the formula for , it’s helpful to write it as a determinant:
The proof of theorem follows from Taylor’s series of two variables function which is given at the end of this section.
EXAMPLE4
Classify the critical points of .
Solution. Suppose is a critical point of
, then
,
.
Solving the above equation for ,
, we get three critical points
,
and
. Look at the Figure 6 and Figure 7. It is clear that
has two local maximum and one saddle point.
Figure 6 Figure 7
Now let us use the second derivative test to classify these critical points. We have
,
and
.
At the critical point ,
and
. That means
. Hence
is a saddle point.
At the critical point ,
and
. That means
and
. Hence
is a local minimum.
At the critical point ,
and
. That means
and
. Hence
is also a local minimum. ■
http://matrix.skku.ac.kr/cal-lab/m-Sec13-8-Exm-4.html
[CAS] http://sage.skku.edu/ 또는 https://sagecell.sagemath.org/
var('x,y')
f(x,y)=x^4+y^4-4*x*y+1
fx=f.diff(x); fy=f.diff(y)
cpoints=solve([fx==0,fy==0],[x,y], solution_dict=True)
for sol in cpoints:
if ((sol[x] in RR) and (sol[y] in RR)):
show([sol[x],sol[y]])
Answer : [−1,−1], [1,1], [0,0]
var('x,y')
f(x,y)=x^4+y^4-4*x*y+1
fxx=diff(f,x,2);fyy=diff(f,y,2);fxy=diff(f,x,y)
a,b=-1,-1
show([fxx(a,b),fyy(a,b),fxy(a,b)])
D=fxx(a,b)*fyy(a,b)-fxy(a,b)^2
print bool(D>0)
print bool(fxx(a,b)>0)
c=f(a,b)
print c #local minimum
print "Since D>0 and $f_{xx}>0$, f(-1,-1)=-1 is a local minimum."
Answer : [12,12,−4], True, True, -1
Since D>0 and ,
is a local minimum.
a,b=0,0
show([fxx(a,b),fyy(a,b),fxy(a,b)])
D=fxx(a,b)*fyy(a,b)-fxy(a,b)^2
TF=bool(fxx(a,b)>0)
print bool(D>0)
print bool(fxx(a,b)>0)
c=f(a,b)
print c
print "Since D<0, (0,0,1) is a saddle point."
Answer : [0,0,−4], False, False, 1
Since D<0, (0,0,1) is a saddle point.
a,b=1,1
show([fxx(a,b),fyy(a,b),fxy(a,b)])
D=fxx(a,b)*fyy(a,b)-fxy(a,b)^2
TF=bool(fxx(a,b)>0)
print bool(D>0)
print bool(fxx(a,b)>0)
c=f(a,b)
print c
print "Since D>0 and $f_{xx}>0$, f(-1,-1)=-1 is a local minimum."
Answer : [12,12,−4], True, True, -1
Since D>0 and ,
is a local minimum.
EXAMPLE5
Find the local extreme values of .
Solution. The domain of is the entire plane and the partial derivatives
and
exist everywhere. Therefore, local extreme values can occur only when
and
.
The only possibility is the origin, where the value of is zero. Since
is never negative, we see that the origin gives a local minimum. (See Figure 8.) This can be written by using Theorem 4. We have
,
and
.
Hence, .
Since and
,
is a point of local minimum. ■
Figure 8 The paraboloid has a local minimum value of
at the origin.
EXAMPLE6
Find the local extreme values of the function
.
Solution. Note that has continuous second order partial derivatives at all points in the plane. The function therefore has extreme values only at the points where
and
are simultaneously zero. This leads to
,
or
.
Therefore, the point is the only point where
may have extreme value. We calculate
.
At ,
and .
Then has a local maximum at
. The value of
at this point is
. ■
EXAMPLE7
Find the local maximum and minimum of
in the domain .
Solution. We have
From the second equation, or
. That is,
or
,
is an integer number.
When , from the first equation,
,
is an integer number. In this case, we have three critical points
and
in the given domain.
And in another case, when , then
. So critical points are
and
in the given domain. We will check that these points are wether a local minimum point or a local maximum point or a saddle point.
Next we compute the second partial derivatives;
and evaluate the discriminant at each critical point;
For , since
,
is a saddle point.
For , since
and
,
is a local minimum.
For , since
,
is a saddle point.
For , since
and
,
is a local minimum.
Therefore has a local minimum at
and
. ■
EXAMPLE8
Find the point on the plane nearest the origin.
Solution. Let be any point on the plane
. Then the distance from the origin to the plane is
.
Since , we obtain
.
Let .
If is minimum value, then
is minimum value. So we get
,
Solving the above equations, we get a critical point .
Next we compute the second partial derivatives;
and evaluate the discriminant at each critical point;
and .
Thus has a local minimum at
. Hence the nearest point in the plane is
. ■
Just see how to implement the user defined function to list all critical points and tabulate along with its type (nature). This appears in the attached sage worksheet.
Absolute Maximum and Minimum Values
For any continuous function defined on a closed interval of finite length
, we know
should have an absolute maximum and an absolute minimum value on
. In this section, we want to determine the analogue of
, a function of two variables
and
.
We will study how to find the absolute maximum and the absolute minimum values of a continuous function on a closed and bounded region in the
-plane.
DEFINITION5 Absolute Maximum and Minimum
(i) If for all
in the domain of
, then
has an absolute maximum at
.
is an absolute maximum value.
(ii)If for all
in the domain of
, then
has an absolute minimum at
.
is an absolute minimum value.
Analogous to have extreme values at endpoint, a function of two variables can have extreme values on the boundary of the closed set .
PROCEDURE
Absolute Maximum and Minimum Values on a Closed Bounded Set
Let be continuous on a closed bounded set
in
. To find absolute maximum and minimum values of
on
,
1. Determine the values of at all critical points in
.
2. Find the maximum and minimum values of on the boundary of
.
3.The greatest function value found in 1 and 2 is the absolute maximum value of on
, and the least function value found in 1 and 2 is the absolute minimum value of
on
.
EXAMPLE9
Find the absolute maximum and minimum values of the on the closed set
.
Solution. To find critical points, we have
and
.
Then is a critical point and so
.
We now determine the maximum and minimum values of on the boundary of
, which is a circle of radius
described by the parametric equations
and
for
.
Substituting and
in terms of
into the function
, we obtain a new function
that gives the values of
on the boundary of
. Thus
The critical points of satisfy
or
. Therefore,
has critical points
and
, which correspond to the points
and
.
The function values are
(critical point)
(boundary point)
(boundary point)
After comparing these values, we can decide that is an absolute minimum value of
and
is an absolute maximum value of
. ■
* Extreme of Implicit function
Let be an implicit function and assume that
has second order continuous partial derivatives.
In order to find extremum of , we will find the point
such that
,
.
Such points are called singular points of . We have
, and
.
If is a point of local maximum or minimum, then
since
.
THEOREM 6
Let be a extreme point and
, then
(i) If , then
has a local minimum at
and
is called a point of local minimum.
(ii) If , then
has a local maximum at
and
is called a point of local maximum.
EXAMPLE10
Find the extreme values of the implicit function
.
Solution. Let .
To find singular points, we must solve the followings;
,
.
Since , we have
. Then two singular points are
.
Since ,
and
. Thus,
,
By Theorem 2, is the local maximum value of
. ■
http://matrix.skku.ac.kr/cal-lab/cal-12-4-Exm1.html
var('x, y');
f(x, y) = x^3 -3*x*y + y^3
p=implicit_plot(f(x, y)==0, (x, -3, 3), (y, -3, 3))
pts=point([(0, 0), (2^(1/3), 4^(1/3))], rgbcolor='red', size=50)
show(p+pts)
Figure 9
Notice that at the tangent is parallel to
axis, but it is not a point of local extremum. Can you see this from the graph? The
is a local maximum.
We have and
.
To classify singular points we need to find the sign of . At
,
. Therefore
is not a point of maximum/minimum value. At
,
. Theorem 6 and the graph shows
is a point of local maximum. The maximum of
is
which occurs at
.
EXAMPLE11
Find the extreme values of the implicit function
.
Solution. Let . Solving the above two equations;
,
we get three singular points
.
Since ,
.
Then we have two extreme points; and
.
For and
, since
,
at
and
.
Hence the implicit function has local maximum value at
. ■
*Expansion of Taylor's Theorem
In this section we shall look at Taylor’s Theorem.
Taylor’s Theorem for function of two variables is as follows.
THEOREM 7
Suppose the function has
order continuous partial derivatives at
. Let
and
be small enough.
Proof. If we let in
,
which is a function of . This single variable function can be expanded as following by Maclaurin formula:
.
Let in the above formula,
.
Differentiate with respect to
,
.
Let in the above equation.
.
Similarly, we can differentiate with respect to
,
.
Let in the above equation.
.
In a similar way, we have a general form;
.
Substituting , we get
. ■
When we replace and
with
and
in Taylor’s Theorem for a two-variable function, it is transformed as follows:
.
Placing ,
in
.
.
This is Maclaurin’s Theorem for two-variable functions which is a special case of the Taylor Theorem.
The case of at
in (Taylor) Theorem 7 is
which is the Mean Value Theorem for the two variable function.
EXAMPLE12
Find the Taylor series for the function at the point
up to degree
.
Solution. ,
,
,
,
,
,
,
.
,
.
,
.
Therefore
. ■
EXAMPLE13
Expand the Taylor series for the function at
.
http://matrix.skku.ac.kr/cal-lab/m-Sec13-8-Exm-13.html
Solution.
x, y=var('x, y');
taylor(x*y^3, (x, 1), (y, -1), 4)
Answer : (y + 1)^3*(x - 1) + (y + 1)^3 - 3*(y + 1)^2*(x - 1) - 3*(y + 1)^2 +3*(y+1)*(x-1)-x+3*y + 3 ■
EXAMPLE14
Plot the surface . Find Taylor's polynomial of degrees 1, 2, 3, and 4. Plot the surface and graph of these polynomial together and show the Taylors polynomial approximates the function as the degree increases.
http://matrix.skku.ac.kr/cal-lab/m-Sec13-8-Exm-14.html
[CAS] http://sage.skku.edu/ 또는 https://sagecell.sagemath.org/
Solution.
var('x,y')
f = cos(x)*cos(y)
a=0
b=0
f1=taylor(f(x,y),(x,a),(y,b),1)
f2=taylor(f(x,y),(x,a),(y,b),2)
f3=taylor(f(x,y),(x,a),(y,b),3)
f4=taylor(f(x,y),(x,a),(y,b),4)
p=plot3d(f(x,y),(x,-3,3),(y,-3,3))
p1=plot3d(f1,(x,-1,1),(y,-1,1),color='red')
p2=plot3d(f2,(x,-1.5,1.5),(y,-1.5,1.5),color='goldenrod')
p3=plot3d(f3,(x,-2,2),(y,-2,2),color='green')
p4=plot3d(f4,(x,-2,2),(y,-2,2),color='violet')
p+p2+p1+p3+p4
Answer :
■
Figure 10
13.8 EXERCISES (Extrema of Multivariate Functions)
http://matrix.skku.ac.kr/Cal-Book/part2/CS-Sec-13-8-Sol.html
1. Let . Find the critical points of
and classify them. Solution. Solve
and
. So we have critical points,
or
. If
, then
. If
, then we have
.
The critical points are
,
,
,
.
Next, we consider second order partial derivatives to get ,
,
.
Then and thus we obtain
at points
,
. This implies that
,
are saddle points.
At points ,
, we observe that
.
Moreover, since ,
has a local maximum
at
. On the other hand, due to
,
has a local minimum
at
.
2. Find the extreme values of the function when
.
Solution.
: critical points
=>
has no local minimum. or maximum at
.
At ,
, so
has a local maximum
at
.
3-4. Locate the maxima, minima, and saddle points of the functions.
3. .
Solution.
http://matrix.skku.ac.kr/cal-lab/cal-12-4-3.html
http://matrix.skku.ac.kr/LA-Lab/ms-1.html
[CAS] http://sage.skku.edu/ 또는 https://sagecell.sagemath.org/
var ('x, y, z')
f(x,y)=2*(x^2-y^2)-x^4+y^4
P=implicit_plot3d (z==f(x, y), (x,-1.5,3/2), (y,-3/2,3/2), (z,-1,1), color= 'goldenrod', opacity=0.6)
P.show()
contour_plot(f, (x,-2,2), (y,-2,2), contours= srange(-2,2,0.25), fill=False, cmap= 'cool', labels=True)
http://matrix.skku.ac.kr/LA-Lab/ms-2.html
[CAS] http://sage.skku.edu/ 또는 https://sagecell.sagemath.org/
var ('x, y, z')
f(x,y)=2*(x^2-y^2)-x^4+y^4
fx=f.diff(x)
fy=f.diff(y)
fxx=diff(f,x,x)
fyy=diff(f,y,y)
fxy=diff(f,x,y)
cpoints=solve([fx==0,fy==0],[x,y],solution_dict=True)
for sol in cpoints:
if ((sol[x] in RR) and (sol[y] in RR) ):
print((sol[x],sol[y]))
(0, 0), (1, 0), (-1, 0), (0, 1), (1, 1), (-1, 1), (0, -1), (1, -1), (-1, -1)
def extreme(f,a,b):
f11=diff(f,x,x)(a,b)
f22=diff(f,y,y)(a,b)
f12=diff(f,x,y)(a,b)
D=f11*f22-f12^2
if(D>0):
if(f11>0):
return "local minimum"
else:
if(f11<0):
return "local maximum"
else:
return "inconclusive"
else:
if(D<0):
return "saddle point"
else:
if(D==0):
return "inconclusive"table = [["Critical Point", "Type"]]
f(x,y)=2*(x^2-y^2)-x^4+y^4
fx=f.diff(x)
fy=f.diff(y)
fxx=diff(f,x,x)
fyy=diff(f,y,y)
fxy=diff(f,x,y)
cpoints=solve([fx==0,fy==0],[x,y],solution_dict=True)
for sol in cpoints:
if ((sol[x] in RR) and (sol[y] in RR)):
a=sol[x].n()
b=sol[y].n()
table.append([(sol[x],sol[y]), extreme(f,a,b)])
html.table(table, header=True)
Critical Point Type
(0, 0) saddle point
(1, 0) local maximum
(-1, 0) local maximum
(0, 1) local minimum
(1, 1) saddle point
(-1, 1) saddle point
(0, -1) local minimum
(1, -1) saddle point
(-1, -1) saddle point
4. .
Solution. Try this on your own.
5. Let
. Answer the following:
(a) Find points of local maximum/minimum and a saddle point when .
(b) Give a condition on for the case when
has only one critical point.
Solution. (a)
(1,1) is a saddle point.
and
and
are points of local minimum.
(b)
If has only one critical point
has a solution and
should not have a solution. So
.
6. Find maximum value of on
.
Solution. ,
.
So the critical point is and thus critical value is
Let ,
,
and .
On , we have
and
,
.
On , we have
and
,
.
On , we have
and
,
.
On , we have
and
.
.
So the maximum value is 2.
7. Find the absolute maximum and minimum of in the domain
which is a closed triangle made of three points (0. 0), (2, 1), (1, 2).
Solution. (1)
critical point :
,
(2) 1. moves on
The absolute maximum
, and the absolute minimum
on
.
2. moves on
The absolute maximum
, and the absolute minimum
on
.
3. moves on
The absolute maximum
, and the absolute minimum
on
.
Hence the absolute maximum is 2 and the absolute minimum is 0.
8. Find the absolute maximum and minimum values on the disk D:
.
Solution. interior of
:
Then implies
If implies
Thus, we get the critical points
If then
This implies .
Critical points are
Thus C
onsider , boundary of
:
so
Moreover, is smallest when
and largest when
But
Thus on D the absolute maximum of is
and the absolute minimum is
9. Find the Taylor series for the function
at the point
.
Solution. ,
,
,
,
,
,
,
,
,
,
,
.
Therefore
.
10. Expand the Maclaurin series for the function .
Solution. ,
,
,
,
,
,
In general,
13.9 Lagrange Multiplier
http://www.youtube.com/watch?v=ry9cgNx1QV8
Constrained Optimization (Lagrange Multiplier)
http://www.slimy.com/~steuard/teaching/tutorials/Lagrange.html
So far we have dealt with finding maximum or minimum of a function of one or more variables without any constraints. Such problems are called unconstrained optimization problems. However, most of real life problems of maximization or minimization involves certain outside conditions or constraints. Such problems are called constrained optimization problems. This section deals with such problems where the constraints are of equality types. The method of Lagrange multipliers is a powerful tool for solving this class of problems without the need to explicitly solve the conditions and use them to eliminate extra variables.
First of all let us consider a minimization/maximization problem of a function of two variables.
Find extreme values of
subjected to
.
Let us look at this problem geometrically. We wish to find the maximum and minimum value of on the unit circle
.
The point at which such a minimum (maximum) occurs is called a minimizer (maximizer).
Note that , for various values of
represent parallels lines. The value of
increases as we move from left to right along the line
. Thus we are looking for a point on the circle
at which the value of
is minimum and maximum.
Look at the Figure 1. It appears that this happens when these curves just touch each other, that is, when they have a common tangent line. In particular, the normal vectors (gradient) to the level curves of and
at the point of maximum or minimum are parallel to each other.
Figure 1
[CAS] http://sage.skku.edu/ 또는 https://sagecell.sagemath.org/
p=contour_plot(f(x,y), (x,-2,2), (y, -2, 2), contours=40, fill=False, cmap='cool', linestyles=['solid'])
p+=implicit_plot(g(x,y)==0, (x,-2,2), (y,-2,2), cmap=['blue'])
p+=points([[sqrt(2)/2,sqrt(2)/2],[-sqrt(2)/2,-sqrt(2)/2]], pointsize = 30, color = 'green')
p+=implicit_plot(f(x,y)==sqrt(2), (x, -2, 2), (y, -2, 2), color= 'red')
p+=implicit_plot(f(x,y)==-sqrt(2), (x, -2, 2), (y, -2, 2), color='red')
p.show(figsize=4, dpi=200)
This turns out to be the necessary condition for a point to be minimum of maximum in general under certain conditions.
Let us consider finding a minimizer of subjected to
.
Suppose is a point of minimum on the level curve
and that the minimum value is
.
Let be a parameterized curve on
with
. Since
has minimum value at
, the composition function
has minimum at
. Hence
.
In particular, is orthogonal to
. We already know that the
is orthogonal to the
. Thus at the point of minimizer
,
is parallel to
provided
.
(We shall see later why do we need .)
More precisely we have the following theorem:
THEOREM 1
Let have continuous first order partial derivatives. Let
be a point of minimizer or maximizer of
subjected to
. Then
and
are parallel to each other. In particular, if
, then there exists a scalar
such that
. Here
is called the Lagrange multiplier.
Let be a point at which
has maximum or minimum subjected to
. Then there exists
such that we have
,
and
.
The are called the Lagrange conditions.
In order to find minimum/maximum of subjected to
, we use the following steps :
Step 1. Find all the points that satisfy the Lagrange conditions .
Step 2. Evaluate at all the points obtained in Step 1. The largest of these values is the maximum value of
and the smallest is the minimum value of
.
Lagrange Conditions in Three Variables
If is a point at which
has maximum or minimum subjected to
with
, then there exists
such that we have
,
,
and
.
EXAMPLE1
Optimize subjected to
.
Solution. Look at the Figure 2 in which level curves of and the curve
are shown. It is clear from the Figure that there are fours points at which the two curves have common tangents. In particular, we can expect to get four points at which
has extreme values.
Figure 2
Using the Lagrange necessary conditions, there exits such that we have the following equations
and
.
From , we have
. This mean
or
.
If , then
. So the points are
.
If , then from the second equation, we have
. Hence from third equation, we get
. Thus the points are
.
We have four possible points of optimizers, and
. Look at the Figure 2. From this figure, it is clear that at these points, the contours of
and
have common tangents. Look at the Figure 3, it is clear that at these points the gradients of
and
are parallel to each other.
Figure 3
The value of at these points are
,
,
. Hence the minimizer occurs
and the minimum value is
. The maximum occurs at two points
with maximum value
. ■
http://matrix.skku.ac.kr/cal-lab/m-Sec13-9-Exm-1.html
[CAS] http://sage.skku.edu/ 또는 https://sagecell.sagemath.org/
var('x,y,lam,t')
f(x,y)=4+x^2-y
g(x,y)=x^2+y^2-1
fx=f.diff(x)
fy=f.diff(y)
gx=g.diff(x)
gy=g.diff(y)
sol=solve([fx==lam*gx,fy==lam*gy,
g==0],x,y,lam,solution_dict=True)
show(sol)
n=len(sol)
for j in range(n):
print f(sol[j][x], sol[j][y])
print "The Solution is tabulated as follows"
html.table([['x, y, (x, y)']] + [(sol[j][x], sol[j][y],
f(sol[j][x], sol[j][y])) for j in range(n)], header = True)
EXAMPLE2
Find the extreme values(maximum and minimum) of subjected to the constrained
.
Solution. Look at the Figure 4 which represent the level curves of and the curve
. It is clear that there are four points at which the two level curves have common tangents. In particular, we can expect to have four points where
has extreme values.
Figure 4
Using the Lagrange conditions, if is a solution of this problem then
and
.
Combining, the first two equations we get . This implies
. Therefore, we have four points
,
,
,
.
The value of the function at the first two points is and the value of the function at the last two points is
. Thus the maximum value of
and occurs at two points
,
. The minimum value of
and it occurs at
,
.
Look at the following Figure 5 which show that the and
are parallel at point of extremes.
Figure 5
http://matrix.skku.ac.kr/cal-lab/m-Sec13-9-Exm-2.html
[CAS] http://sage.skku.edu/ 또는 https://sagecell.sagemath.org/
var('x, y, lam, t')
f(x,y)=x*y
g(x,y)=x^2/8+y^2/2-1
fx=f.diff(x)
fy=f.diff(y)
gx=g.diff(x)
gy=g.diff(y)
sol=solve([fx==lam*gx, fy==lam*gy, g==0], x,y, lam,solution_dict=True)
show(sol)
Answer : [{x:−2, lam:2, y:−1},{x:2,lam:2,y:1},{x:−2,lam:−2,y:1},{x:2,lam:−2,y:−1}]
n=len(sol)
print "The Solution is tabulated as follows"
html.table([["$x$", "$y$", "$f(x, y)$"]] + [(sol[j][x], sol[j][y],
f(sol[j][x], sol[j][y])) for j in range(n)], header = True)
Answer : x y f(x,y)
−2 −1 2
2 1 2
−2 1 −2
2 −1 −2 ■
EXAMPLE3
Find the dimensions of a cylindrical tin (with bottom and top) made up of a metal sheet to maximize its volume such that the total surface area is .
Solution. Let and
be the radius of the base and height of the cylinder. Then the total surface area of the metal sheet is
and the volume is
. Thus mathematically, we can write the above problem as:
maximize subjected to
.
Using the Lagrange conditions, we get
.
After solving we get . Suppose
is a solution of the above system of equation, then further using the constrained we obtain the solution
,
,
,
. ■
EXAMPLE4
Find the points on the ellipsoid that are closest and farthest from the point
.
Solution. If is any point in
, then its distance from
is given by
.
This means we need to optimize subjected to
. Note that
is minimizer (maximizer) of
iff
is minimizer(maximizer) of
. Thus we need to optimize
subjected to
.
Using the Lagrange conditions, we get
,
,
,
.
Substituting the values ,
and
from the first three equations in the fourth, we get
.
Solving the above equation (using sage) for we get
and
.
For , the solution (using sage) is
and .
For , the solution is
and .
http://matrix.skku.ac.kr/cal-lab/m-Sec13-9-Exm-4.html
[CAS] http://sage.skku.edu/ 또는 https://sagecell.sagemath.org/
var('x,y,z,lam')
f=(x-2)^2+(y+1)^2+(z-2)^2
g=x^2+4*y^2+3*z^2-12
fx=f.diff(x)
fy=f.diff(y)
fz=f.diff(z)
gx=g.diff(x)
gy=g.diff(y)
gz=g.diff(z)
cpoints=solve([fx==lam*gx,fy==lam*gy,fy==lam*gy,fz==lam*gz,
g==0],x,y,z,lam,solution_dict=True)
cpoints
for sol in cpoints:
if ((sol[x] in RR) and (sol[y] in RR) and
(sol[z] in RR) and (sol[lam] in RR) and
(sol[lam] in RR)):
show([sol[x].n(),sol[y].n(),sol[z].n(),f(x=sol[x],y=sol[y],z=sol[z]).n()])
■
We should plot the points along with the surfaces.
Now let us consider the following example which explains why do we need, in Theorem 1.
EXAMPLE5
Maximize/Minimize subjected to
.
Solution.
[CAS] http://sage.skku.edu/ 또는 https://sagecell.sagemath.org/
var('x,y,lam')
f(x)=x
g(x,y)=y^2+x^4-x^3
fx=f.diff(x)
fy=f.diff(y)
gx=g.diff(x)
gy=g.diff(y)
solve([fx==lam*gx,fy==lam*gy, g==0],x,y,lam)
Answer : [[x == 1, y == 0, lam == 1]]
that is a solution of the above problem however, Lagrange method fails to detect this point. As
in this case. Look the Figure 6.
http://matrix.skku.ac.kr/cal-lab/m-Sec13-9-Exm-5.html
t=var('t')
cf=contour_plot(f,(x,-.2,1.2),(y,-0.5,0.5),contours=40,fill=false,cmap='cool')
cg=implicit_plot(g,(x,-.2,1.2),(y,-0.5,0.5))
l1=parametric_plot((0,t),(t,-0.5,0.5),color='green')
l2=parametric_plot((1,t),(t,-0.5,0.5),color='red')
show(cf+cg+l1+l2,figsize=5)
Figure 6 ■
Lagrange Multipliers for Two and More Constraints
Here we consider problems of minimization or maximization with two constraints. The method can be generalized to more than two constraints in a similar manner. In particular, we assume that we want to find the maximum and minimum values of a function subject to two constraints of the form
and
. We assume that
and
are non zero and not parallel (that is, they are linearly independent).
Geometrically, this means that we are looking for the extreme values of f that lies on the curve of intersection of the level surfaces and
.
Suppose, is a point on the curve of intersection of the level surfaces
and
and at which
has an extreme value say
. Let us see what this mean in terms of gradients.
Since is a point of extremum of
,
is orthogonal to the tangent vector to the level surface
. On the other hand, we also have
is orthogonal to the level curve
and
is orthogonal to the level curve
. This means that
lies in the plane spanned by two vectors
and
. That is, there exist scalars
and
such that
.
We summarize this in the following theorem (without proof).
THEOREM 2
Let ,
and
be three functions having first order continuous partial derivatives. If
is a solution of the problem
minimize/maximize
subjected to
and
where and
are non zero and parallel. Then there exist scalars
and
such that
.
The scalars and
are called Lagrange multipliers.
Witting the above conditions () along with the constraints, we get the Lagrange conditions
Many time it is continent to define a Lagrangian function
.
Then the Lagrange conditions ( to
) is equivalent to
,
,
and
,
.
EXAMPLE6
The cone is cut by the plane
in some curve
. Find the point on
that is closest to the origin.
This problem can be stated as
minimize
subjected to ,
.
We define the Lagrangian function
Using the necessary condition for optimization we get the following
equations:
.
Now adding to
and
we reduce the above system to
equations in
unknowns;
.
Now we need to solve -
for
,
,
,
.
From we get
and from
we get
.
Equating the two values of and simplifying we get
.
This implies either or
.
(Case I) when , from
, we have
and from , we get
.
Using the above tow equations and solving for we get
which does not have a real solution. Thus
is not possible.
(Case II) when , from
, we have
and from , we get
.
Thus
Solving this equation for we get
. Now
.
Finally
.
Thus the critical points(possible optimizers) are
.
Note that and
.
Hence is a point of maximum.
Hence is a point of minimum. ■
http://matrix.skku.ac.kr/cal-lab/m-Sec13-9-Exm-6.html
x, y, z, lam, mu = var('x, y, z, lam, mu')
f = x^2+y^2 + z^2
g1 = x^2 + y^2 - z^2 - 1
g2 = x + y +z - 4
L = f - g1 * lam - g2*mu
gradL = L.gradient([x, y, z, lam, mu])
cpoints = solve([gradL[0] == 0, gradL[1] == 0, gradL[2] == 0, gradL[3] == 0, gradL[4] == 0],x, y, z, lam, mu, solution_dict=True)
cpoints
n=len(cpoints)
for sol in cpoints:
if ((sol[x] in RR) and (sol[y] in RR) and (sol[z] in RR) and (sol[lam] in RR) and (sol[mu] in RR)):
show([sol[x].n(), sol[y].n(), sol[z].n(), f(x=sol[x],y=sol[y],z=sol[z]).n()])
Answer :
[1.26138721247417,1.26138721247417,1.47722557505167,5.36439079917344]
[6.73861278752583,6.73861278752583,−9.47722557505166,180.635609200827]
13.9 EXERCISES (Lagrange Multiplier)
http://matrix.skku.ac.kr/Cal-Book/part2/CS-Sec-13-9-Sol.html
1. In each of the following problems, find the extreme(maximum and minimum) values of subjected to the given constrained:
(i) subject to
.
(ii) subject to
.
(iii) subject to
.
(iv) subject to
.
(v) subjected to
(vi) subjected to
.
(vii) subjected to
.
Can you generalize this?
2. Find the points on the sphere that are closest and farthest from the point
.
3. Determine the dimensions of a rectangular box, open at the top having volume pf 32 cubic feets and requiring the least amount of material for its construction.
4. Find the maximum and minimum of
on the ellipse given by the intersection of the cylinder and the plane
5. Find the maximum and minimum of
on the ellipse given by the intersection of the cylinder
and the plane
.
6. Find the maximum and ninimum of
when
.
7. In the following exercises find the extreme values of subjected to the two constraints.
(a) ;
;
.
(b) ;
;
.
(c) ;
;
(d) ;
;
.
8. (a) Use Lagrange multipliers to find the highest and lowest points on the ellipse
.
(b) Use Lagrange multipliers to show that the rectangle with maximum area that has a given perimeter is a square.
(c) Use Lagrange multipliers to show that the triangle with maximum area that has a given perimeter is equilateral.
(d) Find the maximum and minimum volumes of a rectangular box whose surface area is $1200 cm^2$ and whose total edge length is $.
(e) Use Sage to ind the maximum of
, subject to the constraint
.
9. Find the absolute maximum value and minimum value of the function
on the ellipse
.
Solution.
Using ,
Substituting in
, then
Substituting in
The absolute maximum value is
The absolute minimum value is
.
10. Find the absolute maximum value of the function on the ellipse
.
Solution.
Using
,
Substituting in
> Substituting
in
Hence and
Now the absolute maximum value of the function is
Calculus
About this book : http://matrix.skku.ac.kr/Cal-Book/
Copyright @ 2019 SKKU Matrix Lab. All rights reserved.
Made by Manager: Prof. Sang-Gu Lee and Dr. Jae Hwa Lee and 김응기. http://matrix.skku.ac.kr/sglee/ and http://matrix.skku.ac.kr/cal-book/
*This research was supported by Basic Science Research Program through the National Research Foundation of Korea (NRF) funded by the Ministry of Education (2017R1D1A1B03035865).